You are on page 1of 40

JUANA COMLEX vs.

FIL-ESTATE
G.R. No. 152272 March 5, 2012
Facts:
Juana Complex I Homeowners Association, Inc. (JCHA), together with individual residents of
Juana Complex I and other neighboring subdivisions (collectively referred as JCHA, et. al.),
instituted a complaint for damages, in its own behalf and as a class suit representing the
regular commuters and motorists of Juana Complex I and neighboring subdivisions who
were deprived of the use of La Paz Road, against Fil-Estate Land, Inc. They claim that the
excavation of La Paz road would not necessarily give rise to a common right or cause of
action for JCHA et al. against tem since each of them has separate and distinct purpose and
each may be affected differently than the others.
Issues:
Whether or not the complaint was properly filed as a class suit
Held:
The necessary elements for the maintenance of a class suit are:
1. the subject matter of controversy is one of common or general interest to
many persons;
2. the parties affected are so numerous that it is impracticable to bring them all to court;
and
3. the parties bringing the class suit are sufficiently numerous or representative of the
class and can fully protect the interests of all concerned.
In this case, the suit is clearly one that benefits all commuters and motorists who use La
Paz Road. The individuals sought to be represented by private respondents in the suit are
so numerous that it is impracticable to join them all as parties and be named individually as
plaintiffs in the complaint.
ROLANDO D. LAYUG v COMMISSION ON ELECTIONS
GR No. 192984 February 28, 2012
Facts:
On March 31, 2010, petitioner Rolando D. Layug (Layug), in his capacity as a taxpayer and
concerned citizen, filed pro se a Petition to Disqualify4 (SPA No. 10-016 [DCN]) Buhay PartyList from participating in the May 10, 2010 elections, and Brother Mike from being its
nominee. He argued that Buhay Party-List is a mere "extension of the El Shaddai," which is
a religious sect. As such, it is disqualified from being a party-list under Section 5, Paragraph
2, Article VI of the 1987 Constitution5, as well as Section 6, Paragraph 1 of Republic Act
(R.A.) No. 79416, otherwise known as the "Party-List System Act."
Issue:
WON the Comelec 2nd division did not issue a notice of promulgation to the petitioners
counsel as required by Rule 13 of the Rules of Court, thereby committing a clear violation of
procedural due process
Held:
A party may sue or defend an action pro se. Under Section 3, Rule 7 of the Rules of Court,
"(e)very pleading must be signed by the party or counsel representing him, stating in either
case his address which should not be a post office box."

It bears stressing that the finality of a decision or resolution is a jurisdictional event which
cannot be made to depend on the convenience of a party. Decisions or resolutions must
attain finality at some point and its attainment of finality should not be made dependent on
the will of a party.
ALLAN C. GO v MORTIMER F. CORDERO
GR No. 164703 May 4, 2010
Facts:
Cordero was an exclusive distributor of a shipping company is Brisbane. After incurring
travel expenses and closing his first deal with a Cordero, he found out that the latter was
directly dealing with the shipping company for the second transaction, cutting off his
commissions. Worse, his lawyers also connived with the client and the shipping company to
take him out of the picture. He sued them all for conspiring in violating his exclusive
distributorship in bad faith and wanton disregard of his rights.
Issue:
WON the trial court erred in overruling Robinsons objection over the improper resort to
summons by publication upon a foreign national like him and in an action in personam
Held:
We find no error committed by the trial court in overruling Robinsons objection over the
improper resort to summons by publication upon a foreign national like him and in an action
in personam, notwithstanding that he raised it in a special appearance specifically raising the
issue of lack of jurisdiction over his person. Courts acquire jurisdiction over the plaintiffs
upon the filing of the complaint, while jurisdiction over the defendants in a civil case is
acquired either through the service of summons upon them in the manner required by law or
through their voluntary appearance in court and their submission to its authority. A party who
makes a special appearance in court challenging the jurisdiction of said court based on the
ground of invalid service of summons is not deemed to have submitted himself to the
jurisdiction of the court.
HYATT INDUSTRIAL MANUFACTURING CORP., and YU HE CHING
vs.
LEY CONSTRUCTION AND DEVELOPMENT CORP., and PRINCETON DEVELOPMENT
CORP
G.R. No. 147143
Facts:
On April 8, 1994, respondent Ley Construction and Development Corporation (LCDC) filed a
complaint for specific performance and damages with the Regional Trial Court of Makati,
Branch 62 (RTC), docketed as Civil Case No. 94-1429, against petitioner Hyatt Industrial
Manufacturing Corporation (Hyatt) claiming that Hyatt reneged in its obligation to transfer
40% of the pro indiviso share of a real property in Makati in favor of LCDC despite LCDCs
full payment of the purchase price of P2,634,000.00; and that Hyatt failed to develop the
said property in a joint venture, despite LCDCs payment of 40% of the pre-construction cost.
Issue:
WON the CA committed grave abuse of discretion and serious errors of law in reversing the
lower courts order dated 3 December 1996 and 21 April 1997, holding respondent nonsuited for failure to enter into pre-trial.

Held:
The truth is that "evidentiary matters" may be inquired into and learned by the parties before
the trial. Indeed, it is the purpose and policy of the law that the parties - before the trial
if not indeed even before the pre-trial - should discover or inform themselves of all the
facts relevant to the action, not only those known to them individually, but also those
known to their adversaries; in other words, the desideratumis that civil trials should
not be carried on in the dark; and the Rules of Court make this ideal possible through the
deposition- discovery mechanism set forth in Rules 24 to 29.
THE CONSOLIDATED BANK AND TRUST CORPORATION (SOLIDBANK),
vs.
DEL MONTE MOTOR WORKS, INC., NARCISO G. MORALES, AND SPOUSE,
GR No. 143338 July 29, 2005
Facts: Respondents defaulted on their monthly installments, the full amount of the loan
became due and demandable pursuant to the terms of the promissory note. Petitioner
likewise alleges that it made oral and written demands upon respondents to settle their
obligation but notwithstanding these demands, respondents still failed to pay their
indebtedness which, as of 09 March 1984, stood at P1,332,474.55.
Issue:
WON the CA gravely erred when it found that private respondents denied the material
allegations of petitioner solidbanks complaint
Held: Rule 33, Section 1, of the Revised Rules of Civil Procedure states the rule on the
effect of judgment on demurrer to evidence. It reads:
SECTION 1. Demurrer to evidence. - After the plaintiff has completed the presentation of his
evidence, the defendant may move for dismissal on the ground that upon the facts and the
law the plaintiff has shown no right to relief. If his motion is denied, he shall have the right to
present evidence. If the motion is granted but on appeal the order of dismissal is reversed he
shall be deemed to have waived the right to present evidence.
SIMEON CABANG, VIRGINIA CABANG and VENANCIO CABANG ALIAS "DONDON",
vs.
MR. & MRS. GUILLERMO BASAY,
GR No. 180587 March 20, 2009
Facts:
Defendant-appellees had been in continuous, open, peaceful and adverse possession of the
same parcel of land since 1956 up to the present. As it turned out, however, when the
Municipality of Molave relocated the townsite lots in the area in 1992 as a big portion of Lot
No. 7778 was used by the government as a public road and as there were many
discrepancies in the areas occupied, it was then discovered that defendant-appellees were
actually occupying Lot No. 7777. On June 23, 1992, plaintiff-appellants filed a Complaint for
Recovery of Property against defendant-appellees.
Issue:
WON the appellate tribunal erred in reversing the judgment of the trial court.
Held: A final and executory judgment may no longer be modified in any respect, even if the
modification is meant to correct erroneous conclusions of fact or law and whether it will be

made by the court that rendered it or by the highest court in the land. The only exceptions to
this rule are the correction of (1) clerical errors; (2) the so-called nunc pro tunc entries which
cause no prejudice to any party, and (3) void judgments.
AMIHAN BUS LINES, INC.,
vs.
ROMARS INTERNATIONAL GASES CORPORATION, represented by CHARLIE J.
SAPUGAY; Regional Trial Court, Branch 36, Iriga City, presided by HON. MILAGROS G.
QUIJANO; and SAMUEL S. SANTAYANA,
GR No. 180819 July 5, 2010
Facts:
On February 20, 2005, at about 2:00 p.m., an almost head-on collision occurred between
respondent Romars gas tanker, with tractor number TCC 583 and trailer number UUP 138,
and petitioners bus, with plate number DVG 844, along Quirino highway in Ragay,
Camarines Sur. The gas tanker was negotiating an inclined curve along Fort Junction Norte,
Ragay, Camarines Sur when it was bumped by an oncoming Amihan Passenger Bus which
suddenly took the lane of the gas tanker. Both vehicles were damaged, but the trailer truck
was a total wreck.
Issue:
WON the gross negligence and incompetence of petitioners former counsel amounts to
extrinsic fraud to justify the annulment of the assailed decision of the respondent RTC
Held:
It is doctrinal that the fraud that will justify annulment of a judgment is extrinsic fraud.
Extrinsic fraud refers to any fraudulent act of the prevailing party in litigation committed
outside of the trial of the case, whereby the defeated party is prevented from fully exhibiting
his side of the case by fraud or deception practiced on him by his opponent, such as by
keeping him away from court, by giving him a false promise of a compromise, or where the
defendant never had the knowledge of the suit, being kept in ignorance by the acts of the
plaintiff, or where an attorney fraudulently or without authority connives at his defeat.
ROXAS & COMPANY, INC.,
vs.
DAMBA-NFSW and the DEPARTMENT OF AGRARIAN REFORM,
G.R. No. 149548 December 4, 2009
Facts:
Roxas & Co. is a domestic corporation and is the registered owner of three haciendas,
namely, Haciendas Palico, Banilad and Caylaway, all located in the Municipality of
Nasugbu, Batangas.
On July 27, 1987, the Congress of the Philippines formally convened and took over
legislative power from the President. This Congress passed Republic Act No. 6657, the
Comprehensive Agrarian Reform Law (CARL) of 1988. The Act was signed by the President
on June 10, 1988 and took effect on June 15, 1988.
Issue:
WON the appellate court erred when it relaxed of the rules by giving due course to DAMBANFSWs appeal in CA G.R. SP No. 72198, the subject of G.R. No. 167845:

Held:
Unlike courts of justice, the DARAB, as a quasi-judicial body, is not bound to strictly observe
rules of procedure and evidence. To strictly enforce rules on appeals in this case would
render to naught the Courts dispositions on the other issues in these consolidated petitions.
BORMAHECO, INCORPORATED,
vs.
MALAYAN INSURANCE COMPANY, INCORPORATED and INTERWORLD BROKERAGE
CORPORATION,
G.R. No. 156599 July 26, 2010
Facts:
For the loss, the Hotel sought indemnity from Malayan under its insurance policy. Malayan
paid the Hotel. In turn, Malayan, which was subrogated to the rights of the Hotel, made
formal demands for reimbursement from Interworld but to no avail. After trial, the RTC
resolved the conflict in favor of the private respondents as it found that the forklift operator
lifted the cargo when it was not yet properly balanced causing it to tilt, fall and sustain
damages.
Issue:
WON the CA committed an error of law when it affirmed in toto the decision of Branch 17,
RTC of Manila.
Held:
Section 4, Rule 10 of the Revised Rules of Court is quite clear with regard to formal
amendments:
SEC. 4. Formal amendments. A defect in the designation of the parties and other clearly
clerical or typographical errors may be summarily corrected by the court at any stage of the
action, at its initiative or on motion, provided no prejudice is caused thereby to the adverse
party.
G.R. No. 178607 : December 5, 2012
DANTE LA. JIMENEZ vs HON. EDWIN SORONGON
Facts: The RTC ordered the issuance of warrants of arrest against the respondents. The
respondents filed an omnibus motion for reconsideration and for deferred enforcement of the
warrants of arrest. The RTC denied the omnibus motion, reiterating that the trial court is
the sole judge on whether a criminal case should be dismissed or not. CA dismissed outright
the petitioners Rule 65 petition for lack of legal personality to file the petition on behalf of the
People of the Philippines.

Issue: Whether the CA committed a reversible error in dismissing outright the petitioners
Rule 65 petition for certiorari for lack of legal personality to file the petition on behalf of the
People of the Philippines?

Ruling: The petitioner has no legal personality to assail the dismissal of the criminal case.
"Every action must be prosecuted or defended in the name of the real party in interest" "who
stands to be benefited or injured by the judgment in the suit, or by the party entitled to the
avails of the suit."
Interest means material interest or an interest in issue to be affected by the decree or
judgment of the case, as distinguished from mere interest in the question involved. By real
interest is present substantial interest, not a mere expectancy or a consequential interest.
When the plaintiff or the defendant is not a real party in interest, the suit is dismissible.
G.R. No. 196877 : November 21, 2012
ELOISA R. NARCISO, Petitioner, v. ESTELITA P. GARCIA, Respondent.
Facts: Narciso filed a motion to dismiss the complaint, alleging that the RTC had no
jurisdiction over the subject matter of the complaint since it averred facts constitutive of
forcible entry. CA held that, while a motion to lift order of default may be filed at any time
after notice and before judgment, Narciso needed to allege facts constituting fraud, accident,
mistake, or excusable negligence that prevented her from answering the complaint and to
show a meritorious defense or that something would be gained by having the order of default
set aside..
Issue: Whether or not the CA gravely abused its discretion
Held: Narciso had the right to file a motion for reconsideration of the trial courts order
denying her motion to dismiss. No rule prohibits the filing of such a motion for
reconsideration. Only after the trial court shall have denied it does Narciso become bound to
file her answer to Garcias complaint. And only if she did not do so was Garcia entitled to
have her declared in default. Unfortunately, the CA failed to see this point.
G.R. No. 194024 April 25, 2012
Go v. Distinction Properties Development and Construction, Inc.
FACTS: Philip L. Go, Pacifico Q. Lim and Andrew Q. Lim (petitioners) are registered
individual owners of condominium units in Phoenix Heights Condominium developed by the
respondent. In August 2008, petitioners, as condominium unit-owners, filed a complaint
before the HLURB against DPDCI for unsound business practices and violation of the
MDDR, alleging that DPDCI committed misrepresentation in their circulated flyers and
brochures as to the facilities or amenities that would be available in the condominium and
failed to perform its obligation to comply with the MDDR.
ISSUES: Whether PHCC is an indispensable party
HELD: An indispensable party is defined as one who has such an interest in the controversy
or subject matter that a final adjudication cannot be made, in his absence, without injuring or
affecting that interest. It is "precisely when an indispensable party is not before the court
(that) an action should be dismissed. The absence of an indispensable party renders all
subsequent actions of the court null and void for want of authority to act, not only as to the
absent parties but even to those present.
GR no. 174669 September 19 2012
Belle Corp vs. Erlinda de Leon, Et. Al

FACTS: RTC dismissed the Complaint against BELLE for failure to state a cause of action
on the ground that there was no allegation in the Complaint that BELLE was a purchaser in
bad faith. Herein respondents then filed a Motion for Reconsideration. Pending the resolution
of their Motion for Reconsideration of the September 23, 1998 Order, herein respondents
filed a Manifestation/Motion to admit their Amended Complaint and at the same time, the
RTC admitted the Amended Complaint of the plaintiffs-appellants. BELLE filed a "Motion for
Reconsideration or to Dismiss the Amended Complaint" wherein it alleged that the Amended
Complaint still stated no cause of action. RTC reconsidered its Order and lifted the dismissal
against BELLE. Aggrieved by the Order of the RTC, respondents filed an appeal with the
CA.CA, reversed and set aside RTC order.
ISSUE: WON cause of action exists to warrant the right of herein respondents to sue against
BELLE.
RULING : The essential elements of a cause of action are (1) a right in favor of the plaintiff
by whatever means and under whatever law it arises or is created; (2) an obligation on the
part of the named defendant to respect or not to violate such right; and (3) an act or
omission on the part of such defendant in violation of the right of the plaintiff or constituting a
breach of the obligation of the defendant to the plaintiff for which the latter may maintain an
action for recovery of damages or other appropriate relief.
[G.R. NO. 195561] March 14, 2012
GOODLAND COMPANY, INC., PETITIONER, VS. ASIA UNITED BANK, ABRAHAM CO,
ATTY. JOEL T. PELICANO AND THE REGISTER OF DEEDS OF MAKATI CITY,
RESPONDENTS.
Facts: Petitioner repudiated the REMs by claiming that AUB and its officers unlawfully filled
up the blank mortgage forms and falsified the entries therein. Petitioner filed a complaint for
annulment of mortgage before the Regional Trial Court (RTC) of Bian, Laguna.
Respondents moved to dismiss the case, calling the attention of the RTC to petitioners
forum shopping. They argued that the two cases were anchored on the alleged falsification
of the REM as basis for the reliefs sought.
The RTC granted the said motion on March 15, 2007 and dismissed on grounds of forum
shopping and litis pendentia. Said court explained that the injunction case and annulment
case were founded on the same transactions, same essential facts and circumstances, and
raise substantially the same issues. Petitioner appealed both dismissals to the CA, the
separate appeals.CA granted the appeal and reversed the RTCs order Respondents filed a
motion for reconsideration.
Issue: Whether or not there is Lis pendentia
Held: The CA concurred with the RTC that petitioners act of forum shopping was deliberate
and malicious considering that it knowingly filed the case despite the pendency of another
case.
Litis pendentia is a Latin term, which literally means a pending suit and is variously referred
to in some decisions as lis pendens and auter action pendant. As a ground for the dismissal
of a civil action, it refers to the situation where two actions are pending between the same
parties for the same cause of action, so that one of them becomes unnecessary and
vexatious.
E.B. VILLAROSA vs. HON. BENITO August 6, 1999

FACTS: Private respondent filed a Complaint for Breach of Contract and Damages against
petitioner before RTC of Makati allegedly for failure of latter to comply with its contractual
obligation in that, other than a few unfinished low cost houses, there were no substantial
developments therein. Summons, together with the complaint, were served upon the
defendant, but the Sheriff's Return of Service stated that the summons was duly served
"upon defendant E.B. Villarosa & Partner Co., Ltd. thru its Branch Manager on May 5, 1998
at their new office Villa Gonzalo, Nazareth, Cagayan de Oro City, and evidenced by the
signature on the face of the original copy of the summons.
ISSUE:
W/N the trial court acquired jurisdiction over the person of petitioner upon service of
summons on its Branch Manager
HELD:
Service of summons upon the branch manager of petitioner at its branch office at Cagayan
de Oro, instead of upon the general manager at its principal office at Davao City is improper.
When the complaint was filed by Petitioner on April 3, 1998, the 1997 Rules of Civil
Procedure was already in force. Sec. 11, Rule 14 revised the former Section 13, Rule 14 of
the Rules of Court. We rule that the service of summons upon the branch manager of
petitioner atits branch office at Cagayan de Oro, instead of upon the general manager at its
principal office at Davao City is improper.
G.R. No. 171872 June 28, 2010
FAUSTO R. PREYSLER, JR., vs. MANILA SOUTHCOAST DEVELOPMENT
CORPORATION
Facts: 15 January 2002, petitioner Fausto R. Preysler, Jr. (petitioner) filed with the
Municipal Trial Court (MTC) of Batangas a complaint for forcible entry against respondent
Manila Southcoast Development Corporation (respondent). the MTC ruled in favor of
petitioner and ordered respondent to vacate the disputed land and to return the possession
of the land to petitioner. Respondent appealed to the Regional Trial Court (RTC). The
petitioner filed a Motion for Reconsideration. The RTC ruled that petitioners Motion for
Reconsideration was fatally flawed for failure to observe the three-day notice rule.
Issue: Whether or not CA committed grave error in affirming the ruling of the public
respondent that the petitioner had violated the three day notice rule
Held: The three-day notice rule is not absolute. A liberal construction of the procedural rules
is proper where the lapse in the literal observance of a rule of procedure has not prejudiced
the adverse party and has not deprived the court of its authority. Indeed, Section 6, Rule 1 of
the Rules of Court provides that the Rules should be liberally construed in order to promote
their objective of securing a just, speedy and inexpensive disposition of every action and
proceeding.
(G.R. No. 186979) Aug 11, 2010
LIMOS V SPOUSES ODONES
Facts: Private respondents Sps Odones filed a complaint for Annulment of Deed, Title and
Damages against petitioners Limos, Rosa delos Reyes and Sps Delos Reyes before Tarlac
RTC. The complaint alleged that they are the owners of a 940 sq m land by virtue of an
Extrajudicial Succession of Estate and Sale dated Jan 29, 2004, executed by the surviving

heirs of Donata Lardizabal, in whom the original title was vested. After registering the
document of conveyance, they found out that the OCT was cancelled and replaced by a TCT
in the petitioners name. Petitioners were able to secure TCT by virtue of a Deed of Sale
allegedly executed by Donata and husband on April 18, 1972.
Issue:
W/N respondents failure to respond to the Request for Admission amounted to an implied
admission & a preliminary hearing should therefore be conducted
Held: No. Non-joinder of indispensable parties is not a ground for dismissal of an action.
The remedy is to implead the non-party claimed to be indispensable. Parties may be added
by order of the court on motion of the party or on its own initiative at any stage of the action
and/or such times as are just. It is only when the plaintiff refuses to implead an indispensable
party despite the order of the court, that the latter may dismiss the complaint. In this case, no
such order was issued.
G.R. No. 172393 Oct 20, 2010
BANK OF COMMERCE, vs. HON. ESTELA PERLAS-BERNABE
Facts: Petitioner sought the consolidation of (Certiorari Petition) with the Receivership Case
and, for that purpose, filed a Motion to Consolidate before RTC Makati. The RTC, however,
denied the motion.
Petitioner elevated the matter to the CA via a Rule 65 petition ascribing grave abuse of
discretion amounting to lack or excess of jurisdiction to the presiding judge in declining to
consolidate the Certiorari Petition with the Receivership Case. CA denied and dismissed the
petition.
Issue: Whether or not Court of Appeals erroneously upheld the trial courts denial of its
motion to consolidate
Held: Elemental is the rule of procedure that the nature of a pleading is to be determined by
the averments in it and not by its title. considering that both the Receivership Case and
the Certiorari Petition have yet to be resolved, we now come to terms with the central issue
of whether the consolidation of these cases is proper under the circumstances. Indeed, the
objectives of judicial economy and simplicity sit well with the prospect of consolidating the
two subject cases.
G.R. No. 185396 Nov 24, 2009
RUFINA FAJARDO vs. ALBERTO COMANDANTE
Facts: Petitioner and her husband filed a complaint against respondent and the Register of
Deeds of Iba, Zambales, before the RTC of Olongapo City. The Fajardos asked the court to
annul the deed of sale that they supposedly executed in Albertos favor, regarding the land in
Barangay Calapacuan, Subic, Zambales, their signatures on the document having been
forged. They also asked the court to cancel Transfer Certificate that the register of deeds
issued in Albertos name based on that deed of sale.
Issue: Whether or not the RTC gravely abused its discretion in not setting aside, as a
consequence, the order that dismissed their case with prejudice.

Held: From the start, the Fajardos have shown a cavalier attitude in moving their case
forward. Although the issues in the case had been long joined by the pleadings of the
parties, they, being the plaintiffs, did not ask the RTC to set it for pre-trial hearing. It
apparently did not alarm them or their counsel at all. They did not oppose it or bother to
check with the court the outcome of the motion that had been set for hearing. Yet they had
ample time to do this since the RTC incurred delay in acting on Albertos motion. The
remedy is not available to them. hat remedy is not available to them.
G.R. No. 165333 Feb 9, 2010
REPUBLIC OF THE PHILIPPINES (DENR) vs. TECHNOLOGICAL ADVOCATES
FOR AGRO-FOREST PROGRAMS ASSOCIATION, INC. (TAFPA, INC.),
Facts: (TAFPA) and DENR represented by its Regional Executive Director (RED) Paet
entered into a contract for community organizing activities, social investigation, and
information education campaign at the San Isidro Tinago Reforestation Sub-Project in Sergio
Osmea, Sr., Zamboanga del Norte.
The OSG filed a Manifestation and Motion asking the RTC to set aside the decision on the
ground of lack of due process. Aggrieved, petitioner sought recourse before the CA via a
petition for Annulment of Judgment. In denying the petition, the CA opined that the RTC
acquired jurisdiction over respondents cause of action.
Issue: CA erred in dismissing the petition for annulment of judgment on the ground that
notice to the deputized counsel was notice to the OSG.
Held: An action to annul a final judgment is an extraordinary remedy, which is not to be
granted indiscriminately by the court.
Under Section 2, Rule 47 of the Rules of Civil Procedure, the only grounds for annulment of
judgment are extrinsic fraud and lack of jurisdiction. Lack of jurisdiction as a ground for
annulment of judgment refers to either lack of jurisdiction over the person of the defending
party or over the subject matter of the claim. It is absence of, or no, jurisdiction; that is, the
court should not have taken cognizance of the petition because the law does not vest it with
jurisdiction over the subject matter.
G.R. No. 151973 Jul 23, 2009
NATIONAL POWER CORPORATION v SPOUSES LORENZO L. LAOHOO
Facts: The trial court issued two Order requiring the petitioner to pay the amount fixed as
just compensation. The dismissal of the petition by the CA was based on the failure to timely
file the petition, such dismissal was not merely based on technicality, but on petitioner's
failure to perfect its appeal on time with the RTC. This Court finds that the petitioners appeal
before the RTC was filed out of time. Petitioner went to the CA alleging grave abuse of
discretion on the part of the trial court in dismissing its notice of appeal.
Issue: Whether or not CA exercise grave abuse of discretion by the RTC in dismissing
petitioner's notice of appeal.
Held: The petitioner cannot invoke the doctrine that rules of technicality must yield to the
broader interest of substantial justice to spare itself from the consequences of belatedly
filing an appeal. While every litigant must be given the amplest opportunity for the proper and
just determination of his cause, free from the constraints of technicalities, the failure to

perfect an appeal within the reglementary period is not a mere technicality. It raises a
jurisdictional problem, as it deprives the appellate court of its jurisdiction over the appeal.
G.R. No. 157107 Nov 24, 2006
ALPINE LENDING INVESTORS v ESTRELLA CORPUZ
Facts: Respondent filed complaints for falsification of private document and estafa against
Zenaida in MTC Caloocan City. Respondent informed Alpine about these developments, but
the latter still refused to turn over the vehicle to her. Alpine submitted to the RTC a motion to
dismiss on the ground that it is not a juridical person, hence, not a proper party in the case
which the RTC denied. Respondent filed her Amended Complaint with an accompanying
Motion to Admit Amended Complaint two (2) days late.
Issue: Whether or ot the trial court erred in admitting respondents amended complaint
Held: The question is not novel. Settled is the rule that a motion to dismiss is not a
responsive pleading for purposes of Section 2, Rule 10. As no responsive pleading had been
filed, respondent could amend her complaint in Civil Case No. C-20124 as a matter of
right. It has always been the policy of this Court to be liberal in allowing amendments to
pleadings in order that the real controversies between or among the parties may be
presented and cases be decided on the merits without delay.
G.R. No. 180045 Nov 17, 2010
GSIS v NLRC
Facts: Respondents were employed as security guards by DNL Security Agency (DNL
Security). By virtue of the service contract entered into by DNL Security and petitioner
Government
Service
Insurance
System,
they
were
ssigned
to
petitioners Tacloban City office, each receiving a monthly income of P1,400.00. Sometime in
July 1989, petitioner voluntarily increased respondents monthly salary to P3,000.00. DNL
Security informed respondents that its service contract with petitioner was terminated. This
notwithstanding, DNL Security instructed respondents to continue reporting for work to
petitioner.
Labor Arbiter (LA) Benjamin S. Guimoc rendered a decision finding that respondents were
not illegally terminated from employment because the employment of security guards is
dependent on the service contract between the security agency and its client.
Issue: Whether or not CA committed a reversible error in finding that the public
respondent NLRC did not commit grave abuse of discretion amounting to lack or
excess of jurisdiction in dismissing the appeal of the petitioner GSIS
Held: Under Section 3, Rule 13 of the Rules of Court, where the filing of pleadings,
appearances, motions, notices, orders, judgments, and all other papers with the
court/tribunal is made by registered mail, the date of mailing, as shown by the post office
stamp on the envelope or the registry receipt, shall be considered as the date of filing. Thus,
the date of filing is determinable from two sources: from the post office stamp on the
envelope or from the registry receipt, either of which may suffice to prove the timeliness of
the filing of the pleadings. If the date stamped on one is earlier than the other, the former
may be accepted as the date of filing.
INTERVENTION
Pascual vs Robles 652 SCRA 573

G.R. No. 182645, June 22, 2011


FACTS:
Respondent questioned the denial of his appeal by filing a petition for review on certiorari
with SC. SC referred the petition to CA for consideration of the merits on the ground that the
said court has jurisdiction concurrent to take cognizance of the said case. CA annulled the
amended RTC decision. An instant petition was later filed assailing the decision of the SC on
the notice of appeal and record on appeal against respondent.
ISSUE: Whether or not intervention is proper in the case at bar.
HELD:
No. In the present case, petitioner was never a party to the proceedings in the RTC and the
CA. This, however, is not an excuse or justification to allow petitioner to file the instant
petition. To do so would put into the hands of the litigants in a case the power to resurrect or
to introduce anew, with the assistance of intervenors, issues to a litigation which have
already been long settled on appeal. Indeed, petitioner may not be allowed to intervene at
this late a stage. Section 2, Rule 19 of the Rules of Court clearly provides that a motion to
intervene may be filed at any time before rendition of judgment by the trial court.
VENUE
Golden Arches Dev't Corp vs St. Francis Square Holdings Inc 640 SCRA 227
G.R. No. 183843 January 19, 2011
FACTS:
Petitioner filed a Motion to Dismiss for lack of cause of action and improper venue. It
claimed that respondent maintained its principal address in Makati as records of the
Securities and Exchange Commission (SEC) in the Cover Sheet of Amended Articles of
Incorporation (wherein it is stated that the business address of ASB Holdings Inc. is at
Makati), Company Relationship Information Sheet, and Director's Certificate dated February
3, 2007 stating that ASB Holdings, Inc., with principal address at Makati, had amended its
Articles of Incorporation by renaming it (ASB Holdings, Inc.) to St. Francis Square Holdings,
Inc., respondent herein, hence, the complaint should have been filed in Makati. By filing the
complaint in Mandaluyong, petitioner concluded that respondent violated Section 2, Rule 4
of the Rules of Court.
ISSUE: Whether or not there is proper venue in the case at bar.
HELD:
Yes. Venue, in essence, concerns a rule of procedure. In personal actions, it is fixed for the
greatest possible convenience of the plaintiff and his witnesses, and to promote the ends of
justice. Respondent's complaint, being one for enforcement of contractual provisions and
recovery of damages, is in the nature of a personal action which, under Section 2, Rule 4 of
the Rules of Court, shall be filed at the plaintiff's residence. Respondent's purpose in filing
the complaint in Mandaluyong where it holds its principal office is obviously for its
convenience and for orderly administration of justice.
JURISDICTION OVER THE RES
Lucas v. Lucas 650 SCRA 667
G.R. No. 190710, June 6, 2011

FACTS:
On July 26, 2007, petitioner filed a Petition to Establish Illegitimate Filiation (with Motion for
the Submission of Parties to DNA Testing) before the RTC of Valenzuela City. The name of
petitioners father was not stated in petitioners certificate of live birth. However, Elsie later
on told petitioner that his father is respondent. Respondent was not served with a copy of the
petition. Nonetheless, respondent learned of the petition to establish filiation.
ISSUE: Whether or not the RTC had jurisdiction over the Respondent.
HELD:
Yes. In an action in personam, jurisdiction over the person of the defendant is necessary for
the court to validly try and decide the case. In a proceeding in rem or quasi in rem,
jurisdiction over the person of the defendant is not a prerequisite to confer jurisdiction on the
court, provided that the latter has jurisdiction over the res. Jurisdiction over the res is
acquired either (a) by the seizure of the property under legal process, whereby it is brought
into actual custody of the law, or (b) as a result of the institution of legal proceedings, in
which the power of the court is recognized and made effective.
REMEDIES
Lina v. CA 135 SCRA 637
G.R. No. L-63397 April 9, 1985
FACTS:
On March 1982, private respondent Northern Motors, Inc. filed with the then Court of First
Instance of Rizal (Pasig) a case for sum of money with damages.
Petitioner filed his answer to the complaint. Respondent court rendered its decision in favor
of plaintiff (herein private respondent). Petitioner filed a motion to set aside decision.
Respondent judge issued an order denying petitioner's motion to set aside decision.
Petitioner filed with the then Court of Appeals a petition for certiorari/prohibition, which was
denied in its decision.
ISSUE: Whether or not the CA erred in the case at bar.
HELD:
No. Under the Rules of Court, the remedies available to a defendant in the Court of First
Instance (now Regional Trial Court) are:
a) The defendant in default may, at any time after discovery thereof and before judgment, file
a motion, under oath, to set aside the order of default on the ground that his failure to
answer was due to fraud, accident, mistake or excusable neglect, and that he has a
meritorious defense; (Sec. 3, Rule 18)
b) If the judgment has already been rendered when the defendant discovered the default,
but before the same has become final and executory, he may file a motion for new trial under
Section 1 (a) of Rule 37;
c) If the defendant discovered the default after the judgment has become final and
executory, he may file a petition for relief under Section 2 of Rule 38; and

d) He may also appeal from the judgment rendered against him as contrary to the evidence
or to the law, even if no petition to set aside the order of default has been presented by him.
(Sec. 2, Rule 41)
Petitioner in this case did not avail himself of any of the above remedies. Instead, he went to
the appellate court on certiorari/prohibition.
TO DETERMINE COUNTERCLAIM
Sandejas v. Ignacio 541 SCRA 61
G.R. No. 155033 December 19, 2007
FACTS:
A unit was leased for the benefit of Benjamin who is the occupant of the unit. The rentals
were paid by Ignacio. The term of the lease is for one (1) year and will expire on October 15,
1994. It appears that Arturo was intending to renew the lease contract. As he had to leave for
the U.S., Arturo drew up a check, and wrote on it the name of the payee, Dr. Manuel Borja,
but left blank the date and amount.
Alice then inquired from UCPB if Arturo still has an account with them. On getting a
confirmation, she together with Rosita drew up a scheme to recover the P3 million from
Arturo. Respondents filed a verified complaint for recovery of a sum of money and damages
against Security Bank and Trust Company (SBTC) and its officers.
ISSUE: Whether or not the lower courts erred in its decision in the case at bar.
HELD:
No.There is no compelling reason for the Court to disturb the findings of facts of the lower
courts. The trial court's findings are as follows: (1) Rosita failed to establish that there is an
agreement between her and Arturo that the latter will give her one-third of the proceeds of
the sale of the Morayta property; (2) petitioners were not able to establish by clear and
sufficient evidence that the P3,000,000.00 which they took from Arturo when they encashed
the subject check was part of the proceeds of the sale of the Morayta property; (3) Rosita's
counterclaim is permissive and she failed to pay the full docket and filing fees for her
counterclaim.
SERVICE OF PLEADINGS/JUDGMENT
Camper Realty Corp v. Pajo-Reyes 632 SCRA 400
G.R. No. 179543 October 6, 2010
FACTS:
Before the completion of the technical survey of the property or on August 31, 1992, Augusto
sold the bigger portion to Camper petitioner. Augusto retained ownership of the remaining
property. 19 years after Rodolfos co-owners of the property were notified two days after the
notarization of SPA of the forged signatures, Nena, Rodolfos sister-co-owner, filed a
complaint against Augusto and her brothers Rodolfo and Godofredo, Jr. before the RTC of
Davao City. Godofredo, Jr. was impleaded as defendant allegedly because he refused to be
a co-plaintiff.
ISSUE: Whether or not there was proper service of judgment in the case at bar.

HELD:
No. The records show that service via registered mail of the copy of the decision addressed
to petitioner was made on December 28, 2006 on a certain Daisy who, petitioner avers, was
not authorized to receive the copy, she being a mere house helper of petitioners director
Arturo F. Campo. Although petitioners principal office and Campos residence are housed in
the same building, Campos house helper Daisy cannot be considered as a person-in-charge
of petitioners office to consider her receipt of copy of the decision on behalf of petitioner.
Neither can the house helpers receipt suffice as service to Campo, even if he is a member
of petitioners Board of Directors, absent a showing that he had been authorized by
petitioner to accept service.
MOTION FOR RECONSIDERATION
Velasco v. Angeles 630 SCRA 1
A.M. OCA IPI No. 05-2353-RTJ September 6, 2010
FACTS:
The Investigating Justice thus recommended that respondent be reprimanded for her
unauthorized absences on May 3, 2005 and August 3, 2005 and that the charge of
falsification be dismissed. As reflected early on, the Court, in its above-quoted Resolution of
June 5, 2006, adopted the findings of the Investigating Justice and approved his
Recommendation. Respondent filed a Motion for Partial Reconsideration which was denied.
Hence a second motion for reconsideration was lodged.
ISSUE: Whether or not a motion for reconsideration can be granted for the second time.
HELD:
Yes. While as a general rule the Court does not give due course to second motions for
reconsideration, this is not without exceptions, as when there is an extraordinarily persuasive
reason and after an express leave has been obtained, both of which are present in this case.
After a considered, hard look at the case, the Court finds respondents second Motion for
Partial Reconsideration to be impressed with merit.
PRETRIAL
Urma v. Beltran 627 SCRA 373
G.R. No. 180836 August 8, 2010
FACTS:
The case stemmed from a complaint filed by the respondents against the petitioners for
partition, quieting of title, recovery of ownership, and damages over two parcels of land. The
NBI performed the examination and found that the questioned fingerprint was not identical
with the genuine specimen thumbmark. Hence, the NBI concluded that the absolute deed of
sale supposedly executed by Laureano was a spurious document. In its decision, the RTC
ruled in favor of the respondents by declaring them the absolute owners of portions of the
disputed land and ordering the petitioners to vacate said portions.
ISSUE: Whether or not pre-trial proceedings can be brushed aside in the case at bar.
HELD: No. The Court has stated on several occasions that the pre-trial forms part of the
proceedings, and matters dealt with therein may not be brushed aside in the process of

decision-making. Otherwise, the real essence of compulsory pre-trial would be


inconsequential and worthless. With regard to the petitioners argument that they should be
excused from the procedural blunder committed by their former counsel, the Court finds it
bereft of merit.
VALID JUDGMENT
Protacio SCRA Laya Mananghaya & Co. 582 SCRA 417
G.R. No. 168654 March 25, 2009
FACTS:
Petitioner filed before the NLRC a complaint for the non-issuance of petitioners W-2 tax
form and the non-payment of benefits. Petitioner also sought moral and exemplary damages
and attorneys fees. Respondent Mario T. Managhaya was also impleaded in his official
capacity as respondent firms managing partner. Labor Arbiter Eduardo J. Carpio rendered a
decision, ordering respondents to jointly and solidarily pay complainant.
ISSUE: Whether or not a petition for review or motion for reconsideration of a decision of the
court shall be refused due course or denied without stating the legal basis therefor.
HELD:
No. No petition for review or motion for reconsideration of a decision of the court shall be
refused due course or denied without stating the legal basis therefor. Obviously, the assailed
resolution is not a "decision" within the meaning of the Constitutional requirement. This
mandate is applicable only in cases "submitted for decision," i.e., given due course and after
filing of briefs or memoranda and/or other pleadings, as the case may be. The requirement is
not applicable to a resolution denying a motion for reconsideration of the decision. What is
applicable is referring to "motion for reconsideration of a decision of the court."
FINALITY OF JUDGMENTS
Lalican v. Insular Life 597 SCRA 159
G.R. No. 183526 August 25, 2009
FACTS:
In a letter, Insular Life informed Violeta that her claim could not be granted since, at the time
of Eulogios death, the insurance policy had already lapsed, and Eulogio failed to reinstate
the same. Violeta filed with the RTC, a Complaint for Death Claim Benefit. Violeta alleged
that Insular Life engaged in unfair claim settlement practice and deliberately failed to act with
reasonable promptness on her insurance claim. The RTC dismissed her case because she
has failed to establish by preponderance of evidence her cause of action against the
defendant.
ISSUE: Whether or not the Decision of the court can still be reviewed despite having
allegedly attained finality and despite the fact that the mode of appeal that has been availed
of by Petitioner is erroneous.
HELD:
The Petition lacks merit. The RTC Decision assailed in this Petition had long become final
and executory. A judgment becomes "final and executory" by operation of law. Finality
becomes a fact when the reglementary period to appeal lapses and no appeal is perfected
within such period. As a consequence, no court (not even this Court) can exercise appellate

jurisdiction to review a case or modify a decision that has become final. When a final
judgment is executory, it becomes immutable and unalterable.
MODES OF APPEAL
Nazareno v. City of Dumaguete 590 SCRA 110
G.R. No. 177795 June 19, 2009
FACTS:
Petitioners were all bona fide employees of the City Government of Dumaguete. They were
appointed to various positions by City Mayor Felipe Remollo shortly before the end of his
term. Newly elected City Mayor Perdices announced during the flag ceremony held at the
City Hall that he was not recognizing the appointments made by former Mayor Remollo.
Thereafter, the City Administrator issued a Memorandum directing the Assistant City
Treasurer to "refrain from making any disbursements, particularly payments for salary
differential[s]" to those given promotional appointments by former Mayor Remollo. Thus,
petitioners were constrained to file with the RTC a Petition for Mandamus with Injunction and
Damages with Prayer for a Temporary Restraining Order and Preliminary Injunction against
Respondents City Mayor Perdices and City Officers.
ISSUE: Whether or not the modes of appeal were properly filed in the case at bar.
HELD:
No. Even if it is settled that the Court shall treat this as a Petition for Review under Rule 45
of the Revised Rules of Court, it faces another obstacle in giving the same due course since
petitioners erroneously filed the appeal directly with this Court. When a party appeals from a
decision of the RTC directly to this Court via a Petition for Review under Rule 45, it must
only raise questions of law; otherwise, its appeal shall be dismissed.
COMPLAINT
Lazaro v. Brewmaster Intl Inc 628 SCRA 574
G.R. No. 182779 August 23, 2010
FACTS:
Respondent, is a marketing company engaged in selling and distributing beer and other
products of Asia Brewery, Inc. It filed a Complaint for Sum of Money against the petitioner
with the Metropolitan Trial Court (MeTC) of Makati City. Petitioner obtained on credit from
respondent beer and other products. Despite repeated demands, respondents have failed
and refused to pay their aforesaid obligation. The MeTC dismissed the complaint,
ratiocinating that respondent, as plaintiff, failed to meet the burden of proof required to
establish its claim by preponderance of evidence.
ISSUE: Whether or not there is a proper cause of action in the case at bar.
HELD:
Yes. If a complaint does not state a cause of action, then no relief can be granted to the
plaintiff and it would necessarily follow that the allegations in the complaint would not warrant
a judgment favorable to the plaintiff. The basic requirement under the rules of procedure is
that a complaint must make a plain, concise, and direct statement of the ultimate facts on
which the plaintiff relies for his claim. Ultimate facts mean the important and substantial facts

which either directly form the basis of the plaintiffs primary right and duty or directly make up
the wrongful acts or omissions of the defendant.
Fedman Dev't Corp vs Agcaoili 656 scra 354
G.R. No. 165025 August 31, 2011
FACTS:
FDC claims that there was a failure to pay the correct amount of docket fee herein because
the complaint did not specify the amounts of moral damages, exemplary damages, and
attorneys fees; that the payment of the prescribed docket fee by Agcaoili was necessary for
the RTC to acquire jurisdiction over the case; and that, consequently, the RTC did not
acquire jurisdiction over this case.
ISSUE:
Whether or not the RTC has jurisdiction over the case at bar.
HELD:
Yes, the RTC has jurisdiction based on the decision that the filing of the complaint or other
initiatory pleading and the payment of the prescribed docket fee are the acts that vest a trial
court with jurisdiction over the claim. In an action where the reliefs sought are purely for
sums of money and damages, the docket fees are assessed on the basis of the aggregate
amount being claimed.
Heirs of Emiliano San Pedro vs Garcia 591 scra 593
G.R. No. 166988, 2009 July 3
FACTS:
The petitioners filed a Complaint for Nullification of Kasulatan ng Bilihang Tuluyan and
Kasulatan ng Pagkakautang and Restoration of Tenurial Rights Covered by Operation Land
Transfer against respondents before the Provincial Adjudicator of the Department of
Agrarian Reform Adjudication Board (DARAB). On September 1995, the Provincial
Adjudicator rendered a Decision dismissing the complaint. On October 1995, petitioners,
filed a Motion for Extension of Time to file a Motion for Reconsideration. After a year, on
October 1996, respondents filed a Manifestation stating that no motion for reconsideration
was filed by petitioners despite their request for an extension, nor was an appeal interposed
by them. Accordingly, respondents prayed for the issuance of an entry of judgment. Later, on
November 1996, respondents then filed a Motion to Issue Order of Finality.
ISSUE:
Whether or not the CA erred in its decision in the case at bar.
HELD:
No. The SC affirmed the decision made by the CA. In said Decision, the CA ruled that the
failure to perfect an appeal within the reglementary period is not a mere technicality, but is
rather, jurisdictional. The CA pointed out that the Revised Rules of the DARAB itself impose
a fifteen-day reglementary period to appeal. Moreover, notwithstanding that technical rules
may be relaxed in the interest of justice, the CA ruled that the delay of two years in the filing
of the appeal in the case at bar no longer fits the liberality rule.
(default)
SOCIAL SECURITY SYSTEM, vs. CHAVEZ
GR No 151259 Oct.13, 2004
Facts:

Private respondents, spouses Juanito and Agustina Obedencio, filed Civil Case No. 94-211
for Specific Performance at the Regional Trial Court of Misamis Oriental, Cagayan de Oro
City, Branch 18. They prayed that the Social Security System (SSS) be ordered (1) to
cancel the mortgage on the properties of the spouses; (2) to release the documents covering
the said properties; and (3) to pay the spouses moral damages in the sum of P80,000;
litigation expenses in the sum of P5,000; and attorneys fees in the sum of P20,000.
On motion of Atty. Alberto Bacal, counsel of the respondent spouses, respondent judge
issued an Order dated April 18, 1995 declaring petitioner in default and allowed private
respondents to present their evidence ex-parte.
Issue:
whether the default order of the lower court should be lifted, so that substantial justice would
prevail over technical rules.
Held:
Procedural rules are not to be disregarded or dismissed simply because their nonobservance may have resulted in prejudice to a partys substantive rights. Like all rules they
are to be followed, except only when for the most persuasive of reasons they may be
relaxed to relieve a litigant of an injustice not commensurate with the degree of his
thoughtlessness in not complying with the procedure prescribed. Here, the petitioner has not
shown any persuasive reason why he should be exempt from abiding by the rules.
Accordingly, the order declaring the petitioner in default and the denial of the motion to lift
the order of default are juridically unassailable. A party who fails to appear at a pretrial
conference may be non-suited or considered as in default.
(motion to dismiss)
Global Business Holdings vs. Surecomp Software
G.R. No. 173463; October 13, 2010
Facts:
Instead of filing an answer, Global filed a motion to dismiss based on two grounds: (1) that
Surecomp had no capacity to sue because it was doing business in the Philippines without a
license; and (2) that the claim on which the action was founded was unenforceable under the
Intellectual Property Code of the Philippines.
Issue:
whether a special civil action for certiorari is the proper remedy for a denial of a motion to
dismiss
Held:
An order denying a motion to dismiss is an interlocutory order which neither terminates nor
finally disposes of a case as it leaves something to be done by the court before the case is
finally decided on the merits. As such, the general rule is that the denial of a motion to
dismiss cannot be questioned in a special civil action for certiorari which is a remedy
designed to correct errors of jurisdiction and not errors of judgment.
(modes of appeal)
G.R. No. 142525 February 13, 2009
FEDERAL BUILDERS, INC., vs. DAIICHI PROPERTIES AND DEVELOPMENT, INC.,
Facts:
On account of this differing computations in determining the deductive costs, Daiichi
engaged the services of an independent quantity surveyor, Davis Langdo and Seah
Philippines, Inc. (DLS), to conduct a survey of the deductive costs. DLS came out with its

own estimate of the deductive cost in the amount of P68,441,415.58, which is closer to that
submitted by Daiichi.
On 29 June 1999, the Arbitral Tribunal issued an Order denying Daiichis Motion to
Commission an Independent Quantity Surveyor, reasoning that the commissioning of an
independent surveyor was not absolutely necessary, and that the engagement of such
surveyor would only be useful if both parties agreed on such engagement. Daiichi filed a
motion for reconsideration, which was also denied by the Arbitral Tribunal in an Order dated
13 July 1999.
ISSUE: WON Federal should have filed a petition for review under Rule 45 of the 1997
Rules of Civil Procedure instead of a Special Civil Action for Certiorari under Rule 65
HELD:
Petitioner should have filed a petition for review under Rule 45 of the 1997 Rules of Civil
Procedure instead of a Special Civil Action for Certiorari under Rule 65. The proper remedy
of a party aggrieved by a decision of the Court of Appeals is a petition for review under Rule
45, which is not identical to a Petition for Certiorari under Rule 65. Under Rule 45, decisions,
final orders or resolutions of the Court of Appeals in any case, i.e., regardless of the nature
of the action or proceedings involved, may be appealed to this Court by filing a petition for
review, which would be but a continuation of the appellate process over the original case. On
the other hand, a special civil action under Rule 65 is an independent action based on the
specific grounds therein provided and, as a general rule, cannot be availed of as a substitute
for the lost remedy of an ordinary appeal, including that to be taken under Rule 45.
Accordingly, when a party adopts an improper remedy, as in this case, such petition may be
dismissed outright.
(verification)
PNCC SKYWAY v PNCC SKYWAY CORPORATION
GR 171231 Feb 17, 2010
FACTS:
Respondent alleged that the petition was fatally defective due to the lack of authority of its
union president, Rene Soriano, to sign the certification and verification against forum
shopping on petitioner's behalf. It alleged that the authority of Rene Soriano to represent the
union was only conferred on June 30, 2006 by virtue of a board resolution,[10] while the
Petition for Review had long been filed on February 27, 2006. Thus, Rene Soriano did not
possess the required authority at the time the petition was filed on February 27, 2006.
The petitioner countered that the Board Resolution[11] dated June 30, 2006 merely
reiterated the authority given to the union president to represent the union, which was
conferred as early as October 2005.
ISSUE: WON the union president, representing the union, was clothed with authority to file
the petition on February 27, 2006.
HELD: The purpose of requiring verification is to secure an assurance that the allegations in
the petition have been made in good faith; or are true and correct, not merely speculative.
This requirement is simply a condition affecting the form of pleadings, and non-compliance
therewith does not necessarily render it fatally defective. Truly, verification is only a formal,
not a jurisdictional, requirement.
(pleadings)
PTA OF ST. MATHEW CHRISTIAN ACADEMY vs. THE METROPOLITAN BANK and
TRUST CO. G.R. No. 176518 March 2, 2010

Facts: Denivin and Josefina Ilagan were granted a loan by the Metropolitan Bank and Trust
Co. secured by a Real Estate Mortgage over parcels of land. Upon default, an extra judicial
foreclosure was conducted with Metropolitan Bank being the highest bidder and for which a
Certificate of Sale was issued. During the period of redemption, a Writ of Possession was
approved in favour of the Bank. On June 30, 2005, the St. Mathew Christian Academy of
Tarlac, Inc., owned by the mortgagors and therefore included in one of the foreclosed lands,
filed a Petition for Injunction with Prayer for Restraining Order.
Issue: WON St. Mathew Christian Academy of Tarlac, Inc. is really a third person which
cannot be bound by the writ of possession issued by the Court.
Ruling: Section7 of Act No. 3135 explicitly authorizes the purchaser in a foreclosure sale to
apply for a writ of possession. It is settled that the issuance of a writ of possession is a
ministerial duty of the court. The purchaser of the foreclosed property, upon ex parte
application and the posting of the required bond, has the right to acquire possession of the
foreclosed property during the 12-month redemption period. Since petitioners possession of
the subject school premises stemmed from their employment or enrolment contracts with the
school, their right to possess the subject school premises cannot be adverse to that of the
school and of its owners. As such, the petitioners cannot be deemed "third parties". The
proper remedy for the petitioners is a separate, distinct and independent suit, provided for
under Act No. 3135.
AMENDMENTS
Case: Tiu vs. PBCOM
569 Scra 436, August 19, 2009
Facts:
Asian Waters, Inc. represented by Tiu applied for a real estate loan with PBCOM to fund
its purified water distribution business. The loan was guaranteed by collateral over the
property. For the second application of loan, it was unsecured but required all the members
of the Board of Directors of AWRI to become sureties and a Surety Agreement was
executed. A demand made by AWRI to the bank to apply as dacion en pago for their existing
loan but PBCOM denied and sent a letter to Tiu demanding full payment of its obligation.
Remained unheeded pushed them to file a collection against petitioners. Petitioners filed
their Answer saying that they were not personally liable because of the Surety Agreement in
their capacities as officers. Founded on the two copies of the agreement, the words " In his
personal capacity" was not stated. PBCOM invoking a liberal interpretation of the Rules,
emphasized that the motion incorporated in the pleading can be treated as a motion for
leave of court to amend and admit the amended complaint pursuant to Sec 3, Rule 10 of
Rules of Court and RTC granted the motion. On appeal, CA dismissed the petition for lack
of merit
Issue:
Whether or not CA committed a reversible error in affirming the Order of the RTC allowing
the substitution of the document by relying on Section 3, Rule 10 of the Rules Of Court?
Ruling:
There was no fraudulent intent on the part of the PBCOM in submitting the altered surety
agreement. The counsel, having no idea that the altered document was submitted, thereby
necessitating the substitution of the surety agreement with the original in order that the case
be judicially resolved. The substitution of it, finds that the RTC did not err in allowing the
substitution.

PERFECTION OF APPEAL
Case: Miel vs. Malindog
579 Scra 119, February 13, 2009
Facts: Miel, the Engineer II of DPWH Samar, filed with Civil Service Commission for
falsification of official documents, dishonesty, conduct prejudicial to the best interest of the
service and grave misconduct against Malindog where the latter's three PDS's were
contradicting and apparently deceitful. Malindog filed an Answer denying the charges and
averred that they were malicious and pure harassment. After the preliminary investigation,
Malindog was charged with dishonesty and Civil Service Comission Head Office affirmed the
findings and impised the penalty of dismissal from service. On appeal, CA affirmed with
modification the former's Decision.
Issue: Whether or not Malindog's appeal before CA was filed beyond the peeiod allowed for
appeal and should have been therefore dismissed?
Ruling: Under the provisions of Rule 43 of the Rules of Court, the appeal from the
judgments, final orders or resolutions of the CSC shall be taken by filing a verified petitin for
review to CA withing fifteen(15) days from notice of the judgment, final order or resolution.
Respondent filed his appeal beyond the 15-day reglamentary period, from April 29 to June
19. Nonetheless, a delay in the filing of an appeal under exceptional circumstances may be
excused in grounds of substiantial justice and equity. The respondent who found guilty if
dishonesty and imposed a penalty of dismissal from service is a severe punishment because
it blemishes a person's record in government service. It is in the greater interest of justice
that the penalty givenbe meticulously reiewed by CA hence, did not err in giving due course
to respondent's appeal.
CONSOLIDATION OF CASES
Case: Steel Corporation of the Philippines vs. Equitable PCI Bank, Inc.
635 Scra 403, November 17, 2010
Facts:
SCP is a domestic corporation engaged in manufacturing and distribution of cold-rolled
and galvanized steel sheets and coils. During its operations, SCP encountered and suffered
from financial difficulties and as a result, SCP was unable to service its principal payment for
its liabilities. EQuitable PCI Bank accounted for the total liabilities of SCP, filed a creditorinitiated petition. Finding its petition sufficient, Rehabilitation Court issued an Order directing
the stay of enforcement of all claims and SCP did not opposed the petition. A recommended
Rehabilitation Plan was issued and the other creditors of SCP namely: DEG Planters
Development Bank, China Banking Corp., Asiatrust Development Bank and GE Money Bank
Inc., filed their Joint Manifestation and Motion declaring their conformity. RTC promulgated a
Decision approving the Modified Rehabilitation Plan. Several petitions were filed by the
creditors to CA for review. The petitions of SCP and IPFI were eventually consolidated but
was denied by CA because it has also been consolidated with the first case.
Issue:
Whether or not the CA erred in refusing to consolidate the cases pending before it?
Ruling:
Consolidation of cases is proper as expressly authorized under Section1, Rule 31 of
Rules of Court and Rule 3, Sec 3 of the 2002 Internal Rules of the CA. In the instant case, all
four cases involve identical parties, same subject matter and issues. All four arose from the
same decision rendered by the Rehabilitation Court thus, imperative upon the CA to
consolidate the cases.

FINAL JUDGMENT
Case: Victorio vs. Rosete
585 Scra 65, April 16, 2009
Facts:
Victorio, made a rental survey of other commercial establishment. On the basis of the
survey, a 25% rental increase was demanded from Chua and Heirs of Yong Tian who
refused to pay the increase so Victorio filed unlawgul detainer case against both lessees.
MTCC dismissed the complaints and affirmed by RTC but CA reversed the Decision and
order to vacate the premises. The decision of CA became final and executory. Chua and
Yong Tian filed a motion to quash the writs of execution and was ganted by MTCC. MTCC
found that the latter had indeed paid Victorio the increased monthly rental while the case
was pending with the RTC. Victorio assailed the quashal of the writ directly to the Supreme
Court but was dismissed. Chua and Yong Tian remained in possession of the properties. A
subsequent increase was then made by the petitioner and the lessees refused. Thus,
Victorio again filed a case for ejectment against the two. MTC dismissed the case and was
affirmed by RTC. CA reversed the decision. A petition for Review was filed before the
Supreme Court and it was denied. Two motions for execution was filed by Victorio before
MTCC but denied by the respondent, Judge Rosete and declared that the case was
considered "Finally Closed and Terminated". Victorio filed an administrative complaint
against Judge Rosete for his refusal to execute the judgment ordering her lessees to vacate
the premises. OCA recommended that respondent Judge was found guilty of gross
ignorance of the law.
Issue:
Whether or not Judge Rosete's refusal of execution of writ of execution is valid?
Ruling:
Judge Rosete is indeed guilty of gross ignorance of the law. The judgment in the said
cases, appealed to the Court has already become final and executory and with the said
decision, execution should issue a matter of right on motion by Victorio in accordance with
Section 1, Rule 39 of the 1997 Rules of Procedure. By refusing to issue the necessary writ
for the ejectment of lessees, Judge Rosete effectively altered or modified the final and
executory Decision. Once a judgment becomes final and executory and the issuance of a
writ of execution becomes a ministerial duy of the court.
SERVICE OF PLEADINGS
Case: Domingo v. Ca
611 Scra 353, February 2, 2010
Facts:
Julio Domingo allegedly executed a Deed of Absolute Sale over a parcel of land in favor of
Marcelino's wife, Mananghaya. The Domingos filed before RTC a complaint against
Marcelino and Mananghaya for the annulment of the Deed of Absolute Sale because the
deed was forged. RTC held that it was forged and affirmed by the CA. RTC issued a writ of
execution however, Marcelino filed with DAR a petition that he be declated the tenantbeneficiary of the property, reenter and retook possession and found him in contempt. Going
back to his petition to DAR, it was granted. Agapita filed, for the second time, contempt and
was ganted by RTC. A motion for reconsideration was filed by Marcelino but was denied. A
petition for certiorari praying for the issuance of TRO and CA dismissed the petition.
Issue: Whether or not CA gravely abused its discretion in dismissing the respondent's
petition?
Ruling: Section 11, Rule 13 of the Rules of Court states, service and filing of pleadings and
other papers must, whenever practicable, be done personally; if made through other modes,

the party concerned must provide a written explanation as to why the service or filing was
not done personally and in adjudging the plausibility of an explanation, a court shall consider
the importance of the subject matter of the case or the issues involved and the prima facie
merit of the pleading sought to be explunged for violation of Section 11. The requisites are
lacking in the present case hence, dismissing the petition.
CONCLUSIVENESS OF JUDGMENT
Case: Abelita III vs Doria
596 Scra 220, August 14, 2009
Facts:
Petitioner, Abelita III, filed a complaint for damages under Articles 32(4) and (9) of the Civil
Code against P/Supt. Doria and SPO3 Ramirez, alleging that the respondents together with
10 unidentified police officers requested him and his wife to proceed to PNP Headquarters.
When Abelita parked his car in front of their house, SPO3 Ramirez grabbed him, forcibly
took the key to his van, barged into the vehicle and conducted a search warrant without a
warrant and resulted to a seixure of a licensed shotgun. Thereafter, SPI3 Ramirez continued
his search and then produced a .45 caliber pistol which he allegedly found inside the vehicle
and arrested the petitioner and detained him without any appropriate charge. Respondents
alleged that petitioner was charged with illegal pissession of firearms and frustrated murder
due to a shooting incident happenedin a certain barangay. An administrative case was also
filed against Abelita III. RTC rendered Decision dismissing the petition.
Issue:
Whether or not the findings in the administrative case against Abelita III arr conclusive in
this case?
Whether or not res judicata may apply against petitioner's claim for damages?
Ruling:
For res judicata to apply, following requisites must be present:
a. the former judgment or order must be final;
b. it must be a judgment or order on the merits;
c. it must have been rendered by a court having jurisdiction over the subject matter and
the parties; and
d. there must be, between the first and second actions, identity of the parties, of subject
matter and of cause of action.
While the present case and te administrative case are based on the same essential facts
and circumstances, the doctrine of res judicata will not apply. The latter deals with
administrative liability while the case deals with civil liability for damages of police authorities.
There is no identity of causes of action in the case. There is no identity of parties between
the present case and the administrative case. It was filed by Sia lao for the latter case and
Sia Lao is not a party on the present case.Doria in the present case were not party to the
administrative case between Sia Lao and petitioner. In the present case, petitioner is the
complainant against respondents. Hence, while res judicata is not a defense to petitioner's
complaint for damages, respondents cannot be held liable for damages.
INTERVENTION
Case: Office of the Ombudsman vs. Sison
612 Scra 702, February 16,2010
Facts:
Isog Han Samar Movement filed a letter-complaint addressed to then Ombudsman, Hon.
Marcelo accusing Gov. Tan and other public officials of Samar, including Sison, of highly
anomalous transactions entered into by them amounting several millions of pesos. Sison

was the Provincial Budget Officer. The letter-complaint stemmed from audit investigation
which found that various purchases went without proper bidding procedures and
documentations; that calamity funds were expended without a State of Calamity having been
declared by the President; and that purchases for rice, medicines, electric fans and cement
were substially overpriced. The Office of the Ombusman rendered a Decision finding Sison
and several officials guilty of grave misconduct, dishonesty, and conduct prejudicial to the
best interest of the service and dismissing him from service. On appeal, CA reversed and set
aside the devision of Office of the Ombudsman against Sison. The Office of the Ombudsman
filed an Omnibus Motion for intervention and to Admit Attached Motion for Rwconsideration
which was denied by CA
Issue:
Whether or not the Office of the Ombudsman may be allowed to intervene and seek
reconsideration of the adverse decision rendered by CA?
Ruling:
To warrant inervention under Rule 19 of the Rules of Court, two requisites must concur:
the movant has a legal interest in the matter in litigation; and intervention must not unduly
delay or prejudice the adjudication of the rights of the parties, nor should the claim of the
intervenor be capable of being properly decided ina separate proceeding. The Office of the
Ombudsman is not an appropriate party to intervene in the instant case. It must remain
partial and detached and it must be mindful of its role as an adjudicator, not an advocate.
MOTIONS
Case: Delos Reyes vs. Ramnani
621 Scra 254, June 18, 2010
Facts:
The trial Court rendered a Decision in favor of respondent Ramnani and a writ of
execution was issued. The Branch Sheriff conducted a public bidding and auction sale over
the property during which respondent was the highest bidder. Consequently, a certificate of
sale and writ of possesion was executed in her favor that resulted to annotation at the back
of the certificate of title. Respondent filed a motion fir the issuance of an order directing the
sheriff to execute the final cergificate of sale in her favor. Petitioner opposed on the twin
grounds that the subject motionws not accompanied by a notice of hearing and that the trial
court's decision can no longer be executed as it is barred by prescription. RTC granted the
motion. Upon appeal, CA denied the petition.
Issue:
Whether or not the subject motion is defective for lack of notice of hearing?
Ruling:
As a general rule, all written motions should be set for hearing under Sev 4, Rule 15 of
the Rules of Court, exempted from this rule are non-litigous motions or motions which maybe
acted upon the court without prejudicing the rights of the adverse party. Respondent is
entitled to the issuance of the final certificate of sale as a mater of right and petitioner is
powerless to oppose the same hence, the subject motion falls under the class of non-litigous
motions.
AMENDMENTS
Case: Que vs. CA
339 Scra 505, August 31, 2000
Facts: Klaver, private respondent, enteed into a Cintact to Sell with GDREC involvibg a unit
of the Wack Wack Twin Towers. After paying the full purchase price, he executed a

Conditional Deed of Sale over the same unit in favor of petitioner Que. After three years,
Klaver giled a complaint for specific performance and damages against Que before the RTC
for alleged violation of the provisions of their contract, primarily to her failure to pay the full
purchase price and her taking possession of te property without his consent. An amended
complaint was filed by Klaver impleading GDREC and sought to recover damages from them
and for Que to surrender posession of the unit to GDREC, who in turn, should execute an
Absolute Deed of Sale in his favor. Que filed a motion to dismiss on the ground that the
amendment to the original Complaint impleading GDREC as an additional defendant
transformed the case to one cognizble by HULRB, and since the claim against her was
merely incidental, it must be resolved by the HULRB together with the claim against
GDREC. RTC dismissed without prejudice the amended Complaint against GDREC, denied
Que's motion to dismiss, graned Klaver's Motion to File Amended Complaint and admitted
the Amended Complaint solely against Que.
Issue: Whether the trial court is vested with jurisdiction over the case filed by Klaver against
Que despite the inclusion of GDREC in the amended complaint?
Ruling: When Klaver amended his Complaint in impleading GDREC, the determination
whether a different cause of action is introduced by amendments to the complaint, what
must be ascertained is whether the defendants shall be required to answer for a liability or
legal obligation wholly different from stated in the original complaint. Complaint of Klaver
against Que which was within the jurisdiction of the trial court. The first amended complaint
alleged substiantially the same causes of action against Que and a new causes of action
against Que and new causes of action against GDREC. Insofar as the cause of action
directed against Que are concerned, they are still within the jurisdiction of the trial court.
Causes of action against GDREC, HULRB had competence over them puesuant to Sec 1,
PD 1344. At any rate, filing of the first amended Complaint did not result in ousting the trial
court of its jurisdiction over the entire case because it retained jurisdiction over the cause of
filed against Que.
FORUM SHOPPING
Case: Marasigan vs. Chevron Phils., Inc
665 Scra 499, February 8, 2012
Facts: Sps. Marasigan and Chevron entered into a dealership and distributorship agreement
wherein Sps. Marasigan cn purchase with the latter petroleum products on credit. To
complete the agreement, Sps. Marasigan executed a deeds if real estate mortgage over
their properties as collateral. Exceeding the credit line and owed Chevron, Spouses failed to
pay the obligation despite oral and written demands frim Chevron thus, filed a petition for
extrajudicial foreclosure against the real estate mortgages executed by Sos. Marasigan in
favor of Chevron. Chevron was able to forclose all the real estate mortgages on Sos.
Marasigan but the amount ws not sufficient to cover the obligation. Subsequently, Chevron
filed a complaint against Sps. Marsigan before RTC Makati to recover deficiency. On the
other hand, Sps. Marasigan filed complaint before RTC Gumaca for the Declartion of Nullity
and/or Annulment of Foreclosure with damages. RTC issued an order denying the motion to
dismiss and that there was ni forum shopping. RTC Gumaca rendered a decision in favor of
Sps. Marasigan. On appeal, CA reversed and set aside the RTC decision.
Issue: Whether the case is dismissible on the grounds of foeum shopping and litis
pendencia?
Ruling: What is pivotal in determining whether forum shopping exists or not is the vexation
caused to the courts and parties-litigants by a party who asks different courts and/or
administrative agencies to rule on the same or related cases and/or grant the same or
substiantially the same reliefs in the process creating possible conflicting decisions. In the

case at bench, all requisites of litis pendencia are present. The first element, identity of
parties, or atleast representing the same interest in both action, exists. Identity of rights
asserted and reliefs prayed for, the reliefs being founded on the same facts, likewise
subsists. Finally, the presence of the thied element, that the identity of the two cases should
be such that the judgment that may be rendered in one would, amount to res judicata cannot
be disputed either.
PNB vs Adela Sia
G.R. No. 165836 February 18, 2009
Facts:
MIDCOM, the registered owner of a land in dispute, entered into a contract to sell with
Galicias or the said land for 480,000.00 Php. The Galicias have an unpaid balance of
70,000.00 Php. MIDOCOM then sold the land to the petitioners. A TCT was issued to the
respondents despite a TRO issued by the RTC. The RTC ruled in favor of Galicias. The
Galicias entered into a contact of real estate mortgaged with PNB. The respondents
appealed and were denied of it and so as to the Supreme Court. The respondents refilled
the case impleading Sia as an indispensible party. The CA ruled in favor of them.
Issue:
Whether or not the case is barred by res judicata?
Ruling:
In the present case, the first three elements of res judicata are present. As to the fourth
element, it is important to note that the doctrine of res judicata has two aspects: first, bar by
prior judgment which is provided in Rule 39, Section 47 (b) of the Rules of Court
and second, conclusiveness of judgment which is provided in Section 47 (c) of the same
Rule.
Xxx
In this case, conclusiveness of judgment exists because respondents again seek to enforce
their right and title over the same subject matter, the litigated property, basing their claim on
the nullity of the judgment in Civil Case No. 84-27347, for failure to implead them therein as
indispensable parties, which had been overruled by final and executory judgments. The
same question cannot be raised again even in a different proceeding involving the same
parties.
Banco de Oro vs Tansipek
G.R. No. 181235 July 22, 2009
Facts:
JOCI entered into a contract with Duty Free Philippines for the contruction of Duty Free in
Mandaue City. Respondent, the authorized collector, deposited the payment of Duty Free to
his own account in PCIB. JOCI filed a complaint because PCIB allowed the said deposit
despite the said lack of authority of the respondent and refused to pay JOCI the full amount
of the check despite demand. PCIB filed a Motion to Dismiss and a third party complaint to
the respondent. Respondent was declared in default and the Motion for Reconsideration was
denied. Respondent filed a Petition for Certiorari but was dismissed.
Issue:

Whether or not a Motion for Reconsideration a proper remedy for an Order of Default?
Ruling:
Respondent Tansipeks remedy against the Order of Default was erroneous from the very
beginning. Respondent Tansipek should have filed a Motion to Lift Order of Default, and not
a Motion for Reconsideration, pursuant to Section 3(b), Rule 9 of the Rules of Court:
Xxx
A Motion to Lift Order of Default is different from an ordinary motion in that the Motion should
be verified; and must show fraud, accident, mistake or excusable neglect, and meritorious
defenses. The allegations of (1) fraud, accident, mistake or excusable neglect, and (2) of
meritorious defenses must concur.
COCOFED vs Republic G.R. No. 177857-58/G.R. No. 178193. September 4, 2012
Facts:
Upon motion of petitioner, the Court approved the conversion of the sequestrated common
shares of San Miguel Corporation, registered in the name of Coconut Industry Investment
Fund Holding Companies, into 753,843,312 SMC Series 1 Preferred Shares. Oppositor
intervenors anchor their plea for reconsideration.
Issue:
Whether or not the intervenors have the right to intervene to the case?
Ruling:
The right of intervention should be accorded to any one having title to property which is the
subject of litigation, provided that his right will be subsequently affected by the direct legal
operation and effect of the decision, and provided also that it is reasonably necessary for
him to safeguard an interested of his own which no other party on record is interested in
protecting.
Cua vs Wallem G.R. No. 171337. July 11, 2012 Facts: Petitioner filed an action for damages against the respondent due to the latters failure
to observe extraordinary diligence in carrying the cargo resulting to loss of some of it.
Respondent filed a motion to dismiss on the ground of prescription since the suit was not
brought within one year after delivery of the goods. Respondent filed an omnibus motion
withdrawing its motion and adopted Advance Shippings motion to dismiss. It made an
express reservation that it was not waiving the defense of prescription and/or laches in its
Answer should be required by the circumstances. The Court required the defendants to file
an answer. The trial court ruled in favor of the petitioner. On appeal, respondent argued that
the petitioners claim is barred by prescription.
Issue: Whether or not prescription may still be raise on appeal?
Ruling: Section 1, Rule 16 of the Rules of Court enumerates the ground on which a motion
to dismiss a complaint may be based, and the prescription of an action is included as one of
the grounds under paragraph (f). the defendant may either raise the grounds in a motion to
dismiss or plead them as an affirmative defense in his answer. The failure to raise or plead
the grounds generally amounts to waiver, except if the ground pertains to (1) lack of
jurisdiction over subject matter, (2) litis pendencia, (3) res judicata, or (4) prescription. If the
facts supporting any of these four listed grounds are apparent from the pleadings or the

evidence on record, the courts may consider these grounds moto proprio and accordingly
dismiss the complaint.
Mejillano vs Lucillo G.R. No. 154717 June 19, 2009
Facts: Respondent bought a parcel of land from the heirs of Hospico Loterina. When he was
about to enter said property, he discovered that the petitioner was occupying the said lot.
Respondent requested that the petitioner vacate the lot but the latter refused. Respondent
filed an action for recovery of possession of real property. The court ruled in favor of
respondent. Petitioner appealed but failed to file an appeal memorandum and the appeal
was dismissed. Petitioner alleged that his failure to file for memorandum was due to
ignorance of the untimely demise of his former counsel and a notice of appeal is what is only
needed.
Issue: Whether or not the filing of appeal memorandum is mandatory?
Ruling: The rule is clear. It is obligatory on the part of petitioner to file his memorandum on
appeal within fifteen days from receipt of the notice to file the same; otherwise, his appeal
will be dismissed. In Enriquez v. Court of Appeals (396 SCRA 377 [2003]), we ruled: Xxx The
use of the word shall in a statute or rule expresses what is mandatory and compulsory.
Further, the Rule imposes upon an appellant the duty to submit his memorandum. A duty is
a legal or moral obligation, mandatory act, responsibility, charge, requirement, trust, chore,
function, commission, debt, liability, assignment, role, pledge, dictate, office, (and)
engagement. Thus, under the express mandate of said Rule, the appellant is duty bound
to submit his memorandum on appeal. Such submission is not a matter of discretion on his
part. His failure to comply with this mandate or to perform said duty will compel the RTC to
dismiss his appeal. In rules of procedure, an act which is jurisdictional, or of the essence of
the proceedings, or is prescribed for the protection or benefit of the party affected is
mandatory.
Sarrosa vs Dizon G.R. No. 183027 : July 26, 2010
Facts: Spouses Sarrosa obtained a loan from Dizon secured by a real estate mortgage.
Petitioners failed to pay their obligation. Respondent filed a Petition to Sell in Extrajudicial
Foreclosure of Mortgage. Petitioners filed a case for Breach of Contract and TRO praying
that respondent be restrained from proceeding with the scheduled extrajudicial foreclosure.
The public auction proceeded and the respondent became the highest bidder. Petitioners
failed to redeem the property and did not vacate the property. The respondent filed a Petition
for the Issuance of a Writ of Possession.
Issue: Whether or not the court erred in ruling that the matter of consolidation was
procedurally improper.
Ruling: The Court agrees with the finding of the Court of Appeals that the RTC of Paranaque
City, Branch 257 did not gravely abuse its discretion in issuing the Order dated June 29,
2007, denying the motion to consolidate LRC Case No. 05-0047 (Ex Parte Petition for
Issuance of Writ of Possession) with Civil Case No. 02-0335 (Breach of Contract, Damages,
and Accounting). Consolidation becomes only a matter of right when the cases sought to be
consolidated involve similar questions of fact and law, provided certain requirements are
met. As stated by the Court of Appeals, no such similarities exist between the Ex Parte
Petition for a Writ of Possession and civil case for Breach of Contract, damages, and
Accounting. Hence, the RTC had the discretion to deny the consolidation of the two cases.
MARIETTA DUQUE vs. JUDGE CRISOSTOMO L. GARRIDO,
A.M. NO. RTJ-06-2027, February 27, 2009

Facts: In a verified letter-complaint dated February 7, 2006 complainant Marietta Duque


charged respondent, Judge Crisostomo L. Garrido of the Regional Trial Court (RTC), Branch
7, Tacloban City, Leyte, with gross violation of Section 15, Article VIII of the 1987
Constitution for rendering a decision beyond ninety (90) days in Criminal Case No. 2000-10580 entitled People v Reynaldo Caones y Royo Sr., et al.
Issue: WON Judge Garrido may be held liable for the delay in judgment without justifiable
reason.
Held: Under Section 9(1) , Rule 140, as amended by A.M. No. 01-8-10-SC, of the Revised
Rules of Court, undue delay in rendering a decision or order is categorized as a less serious
charge. Under Section 11(B) of the same Rule, the penalty for such charge is suspension
from office without salary and other benefits for not less than one (1) nor more than three (3)
months, or a fine of more than P10,000 but not exceeding P20,000.
Heirs of Emiliano San Pedro vs Garcia 591 scra 593
G.R. No. 166988, 2009 July 3
FACTS:
The petitioners filed a Complaint for Nullification of Kasulatan ng Bilihang Tuluyan and
Kasulatan ng Pagkakautang and Restoration of Tenurial Rights Covered by Operation Land
Transfer against respondents before the Provincial Adjudicator of the Department of
Agrarian Reform Adjudication Board (DARAB). On September 1995, the Provincial
Adjudicator rendered a Decision dismissing the complaint. On October 1995, petitioners,
filed a Motion for Extension of Time to file a Motion for Reconsideration. After a year, on
October 1996, respondents filed a Manifestation stating that no motion for reconsideration
was filed by petitioners despite their request for an extension, nor was an appeal interposed
by them. Accordingly, respondents prayed for the issuance of an entry of judgment. Later, on
November 1996, respondents then filed a Motion to Issue Order of Finality.
ISSUE:
Whether or not the CA erred in its decision in the case at bar.
HELD:
No. The SC affirmed the decision made by the CA. In said Decision, the CA ruled that the
failure to perfect an appeal within the reglementary period is not a mere technicality, but is
rather, jurisdictional. The CA pointed out that the Revised Rules of the DARAB itself impose
a fifteen-day reglementary period to appeal. Moreover, notwithstanding that technical rules
may be relaxed in the interest of justice, the CA ruled that the delay of two years in the filing
of the appeal in the case at bar no longer fits the liberality rule.
DBP v. Family foods manufacturing
GR NO 180458 Jul 30, 2009
FACTS: On Sept 15, 1982, respondent Family Foods, a partnership owned and operated by
Spouses Julianco and Catalina Centeno obtained an industrial loan from DBP. The loan
e\was evidenced by promissory note dated Sept. 15, 1982 and payable in seven years. As
security, spouses Centeno executed a real estate mortgage on their parcel of land in Laguna
and chattel mortgage over the buildings, equipments and machineries therein, in favor of
DBP. Family foods moved for the outright dismissal of the petition on the ground that DBP
did not attach material portions of the record, ie. Promissory notes, real estate and chattel
mortgages which are necessary for a complete determination of the merits of the petition.
They assert that DBP violated Sec. 4, Rule 45 of the Rules of Court.

ISSUE: Whether violation pf sec. 4, rule 45 of the rules of court justifies an outright dismissal
of the petition.
HELD: No. As a general rule, a petition lacking copies of essential pleadings and portions of
the case record may be dismissed, however, this is not absolute. Much discretion is left to
the court to determine the necessity for copies of pleadings and other documents .
A careful perusal of the records of the case shows that the petitioners substantially
complied with the procedural requirements of sec 4 rule 45 of RoCP. Attached to the petition
for review as annexes are legible certificate duplicate originals of the assailed CA decision
and resolution. DBP also attached the pleadings filed before the rtc and the latters decision.
The attachment of the pleadings and of the decisions of the rtc and CA provides sufficient
basis to resolve the instant controversy.
REILLO V. SAN JOSE
GR NO 166393 Jun 18, 2009
FACTS: Spouses Quiterio San Jose and Antonina Espiritu Santo were the original registered
owners of a parcel of land located in Rizal. Quiterio and Antonina had five children, namely,
Virginia, Virgilio, Galicano, Victoria and Catalina. Antonina died on July 1, 1970, while
Quiterio died on October 19, 1976. Virginia and Virgilio are also now deceased. Virginia was
survived by her husband Zosimo Sr. and their seven children, while Virgilio was survived by
his wife Julita Gonzales and children, among whom is Maribeth.
Respondents filed a Reply to Answer with Compulsory Counterclaim with a motion to
dismiss the counter-petition for partition on the ground that petitioners failed to pay the
required docket fees for their counter-petition for partition, but was dismissed by the court.
ISSUE: Whether there was a grave error committed by the trial court in dismissing the
defendants-appellants counter petition for partition.
HELD: No. The rule regarding the payment of docket fees upon the filing of the initiatory
pleading is not without exception. It has been held that if the filing of the initiatory pleading is
not accompanied by payment of docket fees, the court may allow payment of the fee within
reasonable time but in no case beyond the applicable prescriptive or reglementary period.
Navida v Dizon
G.R. No. 166355, May 30 : 2011
FACTS: Beginning 1993, a number of personal injury suits were filed in different Texas state
courts by citizens of twelve foreign countries, including the Philippines. The thousands of
plaintiffs sought damages for injuries they allegedly sustained from their exposure to
dibromochloropropane (DBCP), a chemical used to kill nematodes (worms), while working
on farms in 23 foreign countries. The cases were eventually transferred to, and consolidated
in, the Federal District Court for the Southern District of Texas, Houston Division. The
defendants in the consolidated cases prayed for the dismissal of all the actions under the
doctrine of forum non conveniens.
Issue: Whether or not the RTCs have jurisdiction over the subject matter in these cases.
HELD: Yes. The rule is settled that jurisdiction over the subject matter of a case is conferred
by law and is determined by the allegations in the complaint and the character of the relief
sought, irrespective of whether the plaintiffs are entitled to all or some of the claims asserted
therein. Once vested by law, on a particular court or body, the jurisdiction over the subject
matter or nature of the action cannot be dislodged by anybody other than by the legislature
through the enactment of a law.

LEONARDO S. UMALE vs. ASB REALTY CORPORATION


G.R. No. 181126 Jun 15, 2011
FACTS: A parcel of land was originally owned by Amethyst Pearl Corporation, a company
that is, in turn, wholly-owned by respondent ASB Realty Corporation. ASB Realty served on
Umale a Notice of Termination of Lease and Demand to Vacate and Pay. ASB Realty stated
that it was terminating the lease effective midnight of June 30, 2003. Umale failed to comply
with ASB Realty's demands and continued in possession of the subject premises,
even constructing commercial establishments thereon. Umale contends that ASB is not the
real party in interest, thus, filed a petition to dismiss the case.
ISSUE: Can a corporate officer of ASB Realty (duly authorized by the Board of Directors) file
suit to recover an unlawfully detained corporate property despite the fact that the corporation
had already been placed under rehabilitation?
HELD: The Court resolves the issue in favor of ASB Realty and its officers. There is no
denying that ASB Realty, as the owner of the leased premises, is the real party-in-interest in
the unlawful detainer suit.[51] Real party-in-interest is defined as the party who stands to be
benefited or injured by the judgment in the suit, or the party entitled to the avails of the
suit.[52]
ANAMA V. CA
GR NO 187021 Jan 25, 2012
FACTS: Anama filed before the Respondent Court a complaint for declaration of nullity of
the deed of sale, cancellation of transfer certificate of title, and specific performance with
damages against PSB, the Co Spouses, and the Register of Deeds of Metro Manila, District
II.
The Respondent Court dismissed Anama's complaint and upheld the validity of the
sale between PSB and the Co Spouses on August 21, 1991. Anama appealed, at first, to this
Court, and after failing to obtain a favorable decision, tothe Supreme Court.
ISSUE: Whether the respondents failed to substantially comply with the rule on notice and
hearing when they filed their motion for the issuance of a writ of execution with the RTC.
HELD: No, respondents did not violate the Rules of the Court. Wherefore, the petition for
review is denied. The Court has consistently held that a motion that fails to comply with the
requirements is considered a worthless piece of paper which should not be acted upon. The
rule, however, is not absolute. There are motions that can be acted upon by the court ex
parte if these would not cause prejudice to the other party. They are not strictly covered by
the rigid requirement of the rules on notice and hearing of motions.
Charter Chemical and Coating Corp. vs. Tan
G.R. No. 163891 May 21, 2009
FACTS: Herbert Tan and Amalia Sonsing were employees of Charter Chemical and Coating
Corporations Davao branch (Charter Chemical), were placed under preventive suspension
for their failure to satisfactorily explain the discrepancies in the stock inventory at the Davao
depot warehouse. They were also asked to explain the alleged dishonesty in the punching of
their time cards. Charter Chemical advised Tan that they were being terminated from the
service. Hence, they filed a complaint for illegal dismissal and money claims against Charter
Chemical.
The NLRC dismissed Charter Chemicals appeal for having been filed beyond the 10-day
reglementary period. The NLRC granted the motion for reconsideration and gave due course

to Charter Chemicals appeal. Subsequently, the NLRC dismissed Tan complaint for illegal
dismissal. The Court of Appeals granted Tan petition and ruled that the NLRC acted with
grave abuse of discretion in admitting Charter Chemicals belated appeal.
ISSUE: Whether or not Charter Chemicals appeal was valid.
HELD: NO. Art. 223 of the Labor Code, the governing law on the timeliness of an appeal
from the decisions, awards or orders of the Labor Arbiter, is explicit that the aggrieved party
has 10 calendar days from receipt thereof to appeal to the NLRC. Accordingly, this 10-day
reglementary period to perfect an appeal is mandatory and jurisdictional in nature. The
failure to file an appeal within the reglementary period renders the assailed decision final and
executory and deprives the appellate court of jurisdiction to alter the judgment, much less to
entertain the appeal.
GUASCH V. DELA CRUZ
GR NO 176015 Jun 16, 2009
FACTS: Respondent alleged that petitioner was his neighbor and kumadre. On several
occasions, petitioner transacted business with him by exchanging cash for checks of small
amount without interest.
The trial court denied respondents Motion to Amend finding that counsel for
respondent was inexcusably negligent; hence, it has become final and
executory. Respondent filed a Motion for Reconsideration but the same was denied by the
trial court,
ISSUE: Whether the Court of Appeals erred in holding that the trial court committed grave
abuse of discretion when it denied respondents Motion to Amend.
HELD: No. As a general rule, the statutory requirement that when no motion for
reconsideration is filed within the reglementary period, the decision attains finality and
becomes executory in due course must be strictly enforced as they are considered
indispensable interdictions against needless delays and for orderly discharge of judicial
business.
SAN PEDRO CINEPLEX PROPERTIES V. HEIRS OF ENANO
G.R. No. 190754 : November 17, 2010
FACTS: Respondents Heirs of Enano filed on August 17, 2006 a complaint for quieting of
title with damages against petitioner. On October 20, 2006, petitioner filed a Motion to
Dismiss on the ground that the RTC did not validly acquire jurisdiction over it due to
improper service of summons. It argued that, among other things, there was no observance
of the rule that service of summons on a defendant-corporation must be made upon its
president, general manager, corporate secretary, treasurer or in-house counsel.
Respondents contended, however, that the Officers Return showed that the summons
addressed to petitioner was served upon and received by Jay Orpiada (Orpiada), its
manager. They thus moved to declare petitioner in default for failure to file an Answer within
the reglementary period.
Issue: Whether or not RTC erred in declaring petitioner in default?
Held:Motion to Declare petitioner in Default is procedurally unsound. The policy of the law is
to have every litigant's case tried on the merits as much as possible. Hence, judgments by
default are frowned upon. A case is best decided when all contending parties are able to
ventilate their respective claims, present their arguments and adduce evidence in support
thereof. The parties are thus given the chance to be heard fully and the demands of due

process are subserved. Moreover, it is only amidst such an atmosphere that accurate factual
findings and correct legal conclusions can be reached by the courts.
Fedman Dev't Corp vs Agcaoili 656 scra 354
G.R. No. 165025 August 31, 2011
FACTS: On August 30, 1984, FDC cancelled the contract to sell involving the unit and cut
off the electric supply to the unit. Agcaoili was thus prompted to sue FDC and FSCC in the
RTC for injunction and damages. FDC claims that there was a failure to pay the correct
amount of docket fee herein because the complaint did not specify the amounts of moral
damages, exemplary damages, and attorneys fees; that the payment of the prescribed
docket fee by Agcaoili was necessary for the RTC to acquire jurisdiction over the case; and
that, consequently, the RTC did not acquire jurisdiction over this case.
ISSUE:
Whether or not the RTC has jurisdiction over the case at bar.
HELD:
Yes, the RTC has jurisdiction based on the decision that the filing of the complaint or other
initiatory pleading and the payment of the prescribed docket fee are the acts that vest a trial
court with jurisdiction over the claim. In an action where the reliefs sought are purely for
sums of money and damages, the docket fees are assessed on the basis of the aggregate
amount being claimed. Ideally, therefore, the complaint or similar pleading must specify the
sums of money to be recovered and the damages being sought in order that the clerk of
court may be put in a position to compute the correct amount of docket fees.
D.M. FERRER & ASSOCIATES CORPORATION, vs. UNIVERSITY OF SANTO TOMAS
G.R. No. 189496 Feb 1, 2012
FACTS: This case is about a Petition for Review on Certiorari under Rule 45 of the Revised
Rules of Court. Petitioner assails the Court of Appeals (CA) Resolution promulgated on 26
June 2009 dismissing the formers Petition for Certiorari, and the Resolution dated 3
September 2009 denying the subsequent Motion for Reconsideration. On 25 November
2005, petitioner and University of Santo Tomas Hospital, Inc. (USTHI) entered into a Project
Management Contract for the renovation of the 4th and 5th floors of the Clinical Division
Building, Nurse Call Room and Medical Records, Medical Arts Tower, Diagnostic Treatment
Building and Pay Division Building. On 4 August 2008, Judge Bernelito R. Fernandez of
Branch 97 of the Regional Trial Court (RTC) of Quezon City granted the motion and
dismissed the Complaint insofar as respondent UST was concerned.
ISSUE: Whether the trial court committed grave abuse of discretion when it held that the
Complaint stated no cause of action.
HELD: It is settled that the existence of a cause of action is determined by the allegations in
the complaint. In resolving a motion to dismiss based on the failure to state a cause of
action, only the facts alleged in the complaint must be considered. The test is whether the
court can render a valid judgment on the complaint based on the facts alleged and the
prayer asked for. Indeed, the elementary test for failure to state a cause of action is whether
the complaint alleges facts which if true would justify the relief demanded. Only ultimate facts
and not legal conclusions or evidentiary facts, which should not be alleged in the complaint
in the first place, are considered for purposes of applying the test.
ADA VS BAYLON
G.R. No. 182435. August 13, 2012

FACTS: RTC held that the death of Rita during the pendency of the case, having died
intestate and without any issue, had rendered the issue of ownership insofar as parcels of
land which she claims as her own moot since the parties below are the heirs to her estate.
The RTC regarded Rita as the owner of the said 10 parcels of land and, accordingly,
directed that the same be partitioned among her heirs. Nevertheless, the RTC rescinded the
donation inter vivos in favor of Florante. On July 28, 2006, the RTC issued an Order23 which
denied the motion for reconsideration filed by Florante. On appeal, the CA REVERSED and
SET ASIDE the decision of RTC insofar as they decreed the rescission of the Deed of
Donation. The CA REMANDED to the trial court for the determination of ownership.
ISSUE:
WHETHER OR NOT JOINDER OF ACTION IS A GROUND FOR DISMISSAL?
SC HELD:
Misjoinder of causes of action is not a ground for dismissal. Indeed, the courts have the
power, acting upon the motion of a party to the case or sua sponte, to order the severance of
the misjoined cause of action to be proceeded with separately. However, if there is no
objection to the improper joinder or the court did not motu proprio direct a severance, then
there exists no bar in the simultaneous adjudication of all the erroneously joined causes of
action.

BALBUENA VS SABAY
G.R. No. 154720 Sept 4, 2009
FACTS:
The case originated from a complaint filed on March 11, 1972 by the
petitioners with the RTC for ownership and recovery of possession, with damages, of three
parcels of agricultural land situated in Barrio Kaduldolan Manga, Municipality of Tuburan,
Cebu (the lands). In their complaint, the petitioners alleged that Leoncia Sabay (Leoncia)
originally owned the lands which they acquired via an execution sale in a civil case where
Leoncia was the losing party. The RTC rendered a decision in the petitioners favor. It based
its conclusion that the petitioners have a better right to the lands on the finding that the
petitioners acquired the lands in good faith. To the RTC, the petitioners purchase of the
lands in good faith created a right that is superior to the unrecorded earlier sale of the lands
to David Sabay. the CA reversed the RTC decision on appeal.
ISSUE:
WHETHER OR NOT ISSUES NOT RAISED IN THE LOWER COURT CAN BE
RAISED FOR THE FIRST TIME ON APPEAL?
SC HELD:
The newly-alleged facts obviously give rise to issues that were never raised in
the proceedings before the RTC and the CA, and should not therefore be allowed to be
raised at this stage of litigation. The well-settled rule is that issues or grounds not raised
below cannot be resolved on review by the Supreme Court, for to allow the parties to raise
new issues is antithetical to the sporting idea of fair play, justice and due process.

DANDOY VS COURT OF APPEALS


G.R. No. 150089 Aug 28, 2007
FACTS:
For failure of the parties to arrive at an amicable settlement during the preliminary
conference, trial on the merits ensued. After Lopez completed the presentation of her
evidence, Dandoy, through counsel, moved for the dismissal of the complaint by way of a
Demurrer to Evidence. Dandoy relied on the alleged admission of Lopez that the payment

for the jewelry will be made only after the sale of Dandoys property situated at Bicutan. The
trial court denied the Demurrer to Evidence, and set the case for presentation of Dandoys
evidence. Dandoy filed a motion for reconsideration which was likewise denied. Aggrieved,
Dandoy elevated the matter to the CA through a petition for certiorari under Rule 65, praying
that the RTC Orders be annulled, and the case be dismissed. The CA dismissed the petition.
ISSUE:
WHEN SHOULD DEMURRER OF EVIDENCE BE ISSUED?
SC. HELD:
A demurrer to evidence may be issued when, upon the facts adduced and the
applicable law, the plaintiff has shown no right to relief. Where the totality of plaintiffs
evidence, together with such inferences and conclusions as may reasonably be drawn
therefrom, does not warrant recovery against the defendant, a demurrer to evidence should
be sustained. A demurrer to evidence is likewise sustainable when, admitting every proven
fact favorable to the plaintiff and indulging in his favor all conclusions fairly and reasonably
inferable therefrom, the plaintiff has failed to make out one or more of the material elements
of his case, or when there is no evidence to support an allegation necessary to his claim. It
should be sustained where the plaintiffs evidence is prima facie insufficient for a recovery.

LISAM ENTERPRISES, INC VS BANCO DE ORO UNIBANK


G.R. No. 143264 Apr 23, 2012
FACTS:
On August 13, 1999, petitioners filed a Complaint against respondents for
Annulment of Mortgage with Prayer for Temporary Restraining Order & Preliminary
Injunction with Damages with the RTC of Legaspi City. On May 15, 2000, the trial court
issued the questioned Order denying both the Motion for Reconsideration and the Motion to
Admit Amended Complaint. The trial court further ruled that the Amended Complaint can no
longer be admitted, because the same absolutely changed petitioners cause of action.
Petitioners filed the present petition with this Court, alleging that what are involved are pure
questions of law.
ISSUE:
WHETHER OR NOT THE MOTION TO AMEND THE COMPLAINT IN THIS CASE MAY
ONLY BE ALLOWED WITH LEAVE OF COURT?
SC. HELD:
It should be noted that respondents Lilian S. Soriano and the Estate of
Leandro A. Soriano, Jr. already filed their Answer, to petitioners complaint, and the claims
being asserted were made against said parties. A responsive pleading having been filed,
amendments to the complaint may, therefore, be made only by leave of court and no longer
as a matter of right.

LOY JR VS SMCEU-PTGWO
Nob 24, 2009 G.R. No. 164886
FACTS: Petitioners filed a Complaint with Application for Preliminary Attachment for the
collection of unpaid attorneys fees for the legal services they rendered to respondent San
Miguel Corporation Employees Union-Philippine Transport and General Workers
Organization (SMCEU-PTGWO), herein referred to as the Union. Petitioners filed a Motion
for Summary Judgment. They averred that the case was ripe for Summary Judgment
because there was a judicial admission that legal services were indeed rendered which
resulted to the benefits enjoyed by the workers in the 1992-1995 CBA. The trial court

rendered its Decision granting the motion for summary judgment. The Union appealed to the
Court of Appeals which nullified the RTCs Decision and remanding the case to the trial
court. Petitioners filed a motion for reconsideration but it was denied.
ISSUE:
WHETHER OR NOT THE SUPPLEMENTAL PLEADINGS IN THIS CASE IE
PROPER?
SC HELD:
In the case at bar, the subsequent answer could neither validly amend the first
answer nor result in the withdrawal of the latter. A supplemental pleading states the
transactions, occurrences or events which took place since the time the pleading sought to
be supplemented was filed. A supplemental pleading is meant to supply deficiencies in aid of
the original pleading and not to dispense with or substitute the latter. It does not supersede
the original, but assumes that the original pleading is to stand.

MAKATI STOCKS EXCHANGE VS. CAMPOS


G.R. No. 138814 April 16, 2009
FACTS:
On 14 February 1994, the SICD issued an Order granting respondents
prayer for the issuance of a Temporary Restraining Order to enjoin petitioners from
implementing or enforcing the 3 June 1993 Resolution of the MKSE Board of Directors. The
SICD subsequently issued another Order on 10 March 1994 granting respondents
application for a Writ of Preliminary Injunction, to continuously enjoin, during the pendency of
SEC Case No. 02-94-4678, the implementation or enforcement of the MKSE Board
Resolution in question. On 11 February 1997, the Court of Appeals promulgated its Decision
in CA-G.R. SP No. 38455, granting respondents Petition for Certiorari.
ISSUE:
WHAT IS THE EFFECT OF THE DEFENDANTS FILLING OF MOTION TO DISMISS?
SC HELD: A cause of action is the act or omission by which a party violates a right of
another. A complaint states a cause of action where it contains three essential elements of a
cause of action, namely: (1) the legal right of the plaintiff, (2) the correlative obligation of the
defendant, and (3) the act or omission of the defendant in violation of said legal right. If
these elements are absent, the complaint becomes vulnerable to dismissal on the ground of
failure to state a cause of action. If a defendant moves to dismiss the complaint on the
ground of lack of cause of action, he is regarded as having hypothetically admitted all the
averments thereof.

PALMA VS GALVEZ
G.R. No. 165273 Mar 10, 2010
FACTS:
On July 28, 2003, petitioner Leah Palma filed with the RTC an action for
damages against the Philippine Heart Center (PHC), Dr. Danilo Giron and Dr. Bernadette O.
Cruz, alleging that the defendants committed professional fault, negligence and omission for
having removed her right ovary against her will, and losing the same and the tissues
extracted from her during the surgery; and that although the specimens were subsequently
found, petitioner was doubtful and uncertain that the same was hers as the label therein
pertained that of somebody else. On May 7, 2004, the RTC issued its assailed Order
granting private respondents motion to dismiss. Petitioner filed a motion for reconsideration,
which the RTC denied in its Order dated July 21, 2004.Petitioner is now before us alleging
that the public respondent committed a grave abuse of discretion amounting to lack or
excess of jurisdiction .

ISSUE:
WHETHER OR NOT THE ISSUE OF THE SUMMONS IN THIS CASE IS PROPER?
SC. HELD: We have held that a dwelling, house or residence refers to the place where the
person named in the summons is living at the time when the service is made, even though
he may be temporarily out of the country at the time. It is, thus, the service of the summons
intended for the defendant that must be left with the person of suitable age and discretion
residing in the house of the defendant. Compliance with the rules regarding the service of
summons is as important as the issue of due process as that of jurisdiction.
PHIL HEALTH CORP. PROVIDER INC VS. BIR
G.R. No. 167330 June 12, 2008
FACTS: Petitioner protested the assessment in a letter dated February 23, 2000. As
respondent did not act on the protest, petitioner filed a petition for review in the Court of Tax
Appeals (CTA) seeking the cancellation of the deficiency VAT and DST assessments. On
April 5, 2002, the CTA rendered a decision, the Petition for Review is PARTIALLY
GRANTED. Respondent appealed the CTA decision to the [Court of Appeals (CA)] insofar as
it cancelled the DST assessment. He claimed that petitioners health care agreement was a
contract of insurance subject to DST under Section 185 of the 1997 Tax Code.
ISSUE: WHETHER OR NOT THE JUDGMENT IS FINAL IN THIS CASE?
SC HELD: When a minute resolution denies or dismisses a petition for failure to comply with
formal and substantive requirements, the challenged decision, together with its findings of
fact and legal conclusions are deemed sustained.It is true that, although contained in a
minute resolution, our dismissal of the petition was a disposition of the merits of the case.
When we dismissed the petition, we effectively affirmed the CA ruling being questioned. As a
result, our ruling in that case has already become final. When a minute resolution denies or
dismisses a petition for failure to comply with formal and substantive requirements, the
challenged decision, together with its findings of fact and legal conclusions, are deemed
sustained. But what is its effect on other cases?
NAVARRO VS ERMITA
G.R. No. 180050 Feb 10, 2010
FACTS:
On November 10, 2006, petitioners Rodolfo G. Navarro, Victor F. Bernal and Rene O.
Medina, former political leaders of Surigao del Norte, filed before this Court a petition for
certiorari and prohibition (G.R. No. 175158) challenging the constitutionality of R.A. No.
9355.6 The Court dismissed the petition on technical grounds. Their motion for
reconsideration was also denied. Undaunted, petitioners, as taxpayers and residents of the
Province of Surigao del Norte, filed another petition for certiorari8 seeking to nullify R.A. No.
9355 for being unconstitutional. They alleged that the creation of Dinagat as a new province,
if uncorrected, would perpetuate an illegal act of Congress, and would unjustly deprive the
people of Surigao del Norte of a large chunk of the provincial territory, Internal Revenue
Allocation (IRA), and rich resources from the area. They pointed out that when the law was
passed, Dinagat had a land area of 802.12 square kilometers only and a population of only
106,951, failing to comply with Section 10, Article X of the Constitution and of Section 461 of
the LG.
ISSUE: WHETHER OR NOT THE PARTY ARE THE PROPER PARTY INTEREST?
SC HELD: At that time, movants-intervenors had nothing at stake in the outcome of this
case. While it may be argued that their interest in this case should have commenced upon

the issuance of COMELEC Resolution No. 8790, it is obvious that their interest in this case
then was more imaginary than real. This is because COMELEC Resolution No. 8790
provides that should the decision in this case attain finality prior to the May 10, 2010
elections, the election of the local government officials stated therein would only have to be
postponed.

NAVIDA VS DIZON, JR.


G.R. No. 125078 May 30, 2011
FACTS:
Beginning 1993, a number of personal injury suits were filed in different Texas
state courts by citizens of twelve foreign countries, including the Philippines. The thousands
of plaintiffs sought damages for injuries they allegedly sustained from their exposure to
dibromochloropropane (DBCP), a chemical used to kill nematodes (worms), while working
on farms in 23 foreign countries. The cases were eventually transferred to, and consolidated
in, the Federal District Court for the Southern District of Texas, Houston Division. The
defendants in the consolidated cases prayed for the dismissal of all the actions under the
doctrine of forum non conveniens.
ISSUE:
WHETHER OR NOT THERE IS A JURISDICTION OVER THE DEFENDANTS IN THIS
CASE?
SC HELD:
The RTC of General Santos City and the RTC of Davao City validly acquired
jurisdiction over the persons of all the defendant companies. All parties voluntarily,
unconditionally and knowingly appeared and submitted themselves to the jurisdiction of the
courts a quo. All the defendant companies submitted themselves to the jurisdiction of the
courts a quo by making several voluntary appearances, by praying for various affirmative
reliefs, and by actively participating during the course of the proceedings below.
Sy Tiovng Shiou vs. Sy Chim
G.R. No. 174168 Mar 30, 2009
Facts:
On 30 May 2003, four criminal complaints were filed by Sy Chim and Felicidad Chan Sy
(Spouses Sy) against Sy Tiong Shiou, Juanita Tan Sy, Jolie Ross Tan, Romer Tan, Charlie
Tan and Jessie James Tan (Sy Tiong Shiou, et al.) before the City Prosecutors Office of
Manila. The cases were later consolidated. In these complaints, the Spouses Sy averred
that they are stockholders and directors of Sy Siy Ho & Sons, Inc. (the corporation) who
asked Sy Tiong Shiou, et al., officers of the corporation, to allow them to inspect the books
and records of the business on three occasions to no avail. In a letter4 dated 21 May 2003,
Sy Tiong Shiou, et al. denied the request, citing civil and intra-corporate cases pending in
court.
ISSUE:
WHETHER OR NOT THE THIRD PARTY COMPLAINT IS PROHIBITED UNDER RULES
OF PROCEDURE FOR INTERIM CORPORATE CONTROVERSIES?
SC HELD:
A third-party complaint is a claim that a defending party may, with leave of
court, file against a person not a party to the action, called the third-party defendant, for
contribution, indemnity, subrogation or any other relief, in respect of his opponents claim. It
is actually a complaint independent of, and separate and distinct from the plaintiffs
complaint. In fact, were it not for Rule 6, Section 11 of the Rules of Court, such third-party

complaint would have to be filed independently and separately from the original complaint by
the defendant against the third-party defendant.
MARIETTA DUQUE vs. JUDGE CRISOSTOMO L. GARRIDO,
A.M. NO. RTJ-06-2027, February 27, 2009
Facts:
In a verified letter-complaint dated February 7, 2006 complainant Marietta Duque charged
respondent, Judge Crisostomo L. Garrido of the Regional Trial Court (RTC), Branch 7,
Tacloban City, Leyte, with gross violation of Section 15, Article VIII of the 1987 Constitution
for rendering a decision beyond ninety (90) days in Criminal Case No. 2000-10-580 entitled
People v Reynaldo Caones y Royo Sr., et al.
Issue: WON Judge Garrido may be held liable for the delay in judgment without justifiable
reason.
Held:
Likewise, the Code of Judicial Conduct under Rule 3.05 of Canon 3 dictates as follows:
Rule 3.05 A judge shall dispose of the court's business promptly and decide cases within
the required periods. Respondent Judge Garrido clearly violated both the Constitution and
the Code of Judicial Conduct when he failed to decide Criminal Case No. 2000-10-580 within
the 90-day period to decide cases prescribed for the lower courts. Whenever a judge cannot
decide a case promptly, all he has to do is to ask the Court for a reasonable extension of
time to resolve it. In this case, granting that it was for a justifiable reason to render a decision
or resolve a matter beyond the reglementary period, the respondent could have sought
additional time by simply filing a request for extension. Respondent, however, did not avail of
such relief.

You might also like